Nothing Special   »   [go: up one dir, main page]

Workbook in MOI

Download as pdf or txt
Download as pdf or txt
You are on page 1of 80

 

      
 
 
 
UNIVERSITY OF SOUTHERN MINDANAO 
COLLEGE OF BUSINESS, DEVELOPMENT ECONOMICS AND 
MANAGEMENT 
KABACAN, COTABATO 
 
 
 

WORKBOOK 
 

IN 
 
 

MATH OF INVESTMENTS 
 
 
 
 
 
 
 
 
 

ACCOUNTANCY DEPARTMENT 
 
 
 
 
Compiled by: 
REINZON JAMES G. JUANITEZ 
RESTY MYRRH B. PURCA 
ELLA MAE P. VILLANUEVA 
 
 
2019 Edition  
 
 
No part of this book may be reproduced or utilized in any form or by any means, electronic or mechanical,  
including photocopying, recording, or any information storage and retrieval system,  
without permission in writing from the University of Southern Mindanao. 
 
 
 
 
  
About the University of Southern Mindanao 
 
 
USM Mission 
 
Help  accelerate  the  socio‐economic  development,  promote  harmony  among  the  diverse 
cultures in Southern Philippines, and improve the quality of life through instruction, research, 
extension and production. 
 
 
 
 
 
USM Vision 
 
Quality and relevant education for its clientele to be globally competitive, culture sensitive 
and morally responsive human resources for development. 
 
 
 
 
 
USM Core Values 
 
Goodness 
Responsiveness 
Excellence 
Assertion of Right and 
Truth 
 
 
 
 
 
 
 
 
 
 
 
 
 
 
 
 
 
 
 
Workbook in Math of Investments    ii 
 
 

 
USM Quality Policy Statement 
 
The University of Southern Mindanao, as a premier university, is committed to provide 
quality instruction, research development and extension services and resource generation 
that exceed stakeholders’ expectations through the management of continual 
improvement efforts on the following initiatives. 
1. Establish key result areas and performance indicators across all mandated 
functions; 
2. Implement quality educational programs; 
3. Guarantee competent educational service providers; 
4. Spearhead need‐based research outputs for commercialization, publication, 
patenting, and develop technologies for food security, climate change mitigation 
and improvement in the quality of life; 
5. Facilitate transfer of technologies generated from research to the community for 
sustainable development; 
6. Strengthen relationship with stakeholders; 
7. Sustain good governance and culture, sensitivity; and Comply with customer, 
regulatory and statutory requirements. 
 
 
USM Institutional Outcomes 
 
The graduates of USM shall: 
1. provide leadership in various development programs both public and private, 
2. be equipped with technical, conceptual and human resource skills, 
3. engage in entrepreneurial activities, 
4. be able to adapt to diverse culture, and 
5. pursue advanced studies in emerging related fields. 
   

Workbook in Math of Investments    iii 
 
 

 
About the College of Business, Development  
Economics and Management 
 
 
 
CBDEM Goal 
 
To produce competent and responsible professionals who will provide leadership in 
business, development economics and management. 
 
 
CBDEM Objectives 
 
The  College  of  Business,  Development  Economics  and  Management  particularly  the 
Department of Accountancy aims to: 

1. Develop professionally competent and socially responsible graduates equipped with 
theoretical  knowledge,  sound  principles  &  techniques  to  serve  the  needs  of  the 
increasingly complex, rapidly changing global business environment and aware of the 
profound magnitude of their role in nation building and in the stewardship of business 
resources; 
2. Provide adequate training to prepare graduates for the various types of assessments 
(including  professional  licensures  and  certifications)  and  career  as  professional 
accountants  in  public  practice,  commerce  and  industry,  government  and  academe 
and for entrepreneurship; 
3. Produce  competent  professional  accountants  with  appropriate  values,  ethics  and 
attitudes  capable  of  making  a  positive  contribution  over  their  lifetimes  to  the 
profession and society; and 
4. Develop  the  qualities  that  enhance  the  students’  professional  and  research 
competence  to  undertake  theoretical  and  empirical  researches  in  the  various 
disciplines of accounting that will advance the frontiers of knowledge in these fields 
that will promote the development of the nation. 
 

Workbook in Math of Investments    iv 
 
 

 
PREFACE 
 
This workbook in Mathematics of Investment is designed for USM Accountancy and 
Management  Accounting  students  that  take  up  the  course  subject  ACC  112  –  Math  of 
Investments.  This  workbook  covers  the  topics  on  simple  interest  and  bank  discount, 
compounding  or  future  value  of  amounts,  discounting  or  present  value  amounts,  simple 
ordinary  annuity,  simple  annuity  due,  simple  deferred  annuity,  simple  perpetuity,  sinking 
fund,  amortization,  depreciation,  stocks  and  bonds,  and  complex  or  general  annuity‐  all  of 
which are tools in preparation for the higher finance/accounting/business subjects. 
 
Each  chapter  includes  brief  discussion  of  the  topics,  illustration  problems,  and 
exercise sheets for students to practice what they’ve learned on the chapter. If supplemented 
with discussion materials and similar discussions that are available, the students will develop 
strong conceptual and computational background in mathematics of investment which will 
help  them  develop  critical  thinking  for  the  business  organization.  Proficiency  to  the  topics 
presented  in  this  workbook  will  be  attained  if  the  students  be  guided  and  facilitated  well, 
especially in this no face‐to‐face learning. 
 
  Computations  are  made  with  the  use  of  tables  and  formulas.  And  step‐by‐step 
explanation  of  the  calculations  are  also  presented  for  illustration.  Also,  the  students  are 
encouraged to use both the basic (ordinary) and scientific calculators while taking this course 
subject in preparation for the actual board examination.  
 
  May you enjoy the journey towards working with the growth and decline of money, 
preparation  of  different  table  for  financial  accounting  and  reporting  purposes,  and  the 
knowledge on how stocks and bonds work through the financial market. 
 
   
 
 
REINZON JAMES JUANITEZ 
RESTY MYRRH PURCA 
ELLA MAE VILLANUEVA 
 
 
 
   

Workbook in Math of Investments    v 
 
 

 
TABLE OF CONTENTS 

CHAPTER 1. SIMPLE INTEREST AND BANK DISCOUNT 

    Simple Interest 
      Definition of Terms              1 
      Formulas                1 
      Manipulating the Simple Interest Formula        3 

    Bank Discount 
      Definition of Terms              5 
      Formulas                4 

    Promissory Notes 
      Definition of Terms              6 
      Formulas                8 
      Discounting of Notes             9 
    Exercises                11 

CHAPTER 2. COMPOUND AMOUNT OR FUTURE VALUE 

    Time Value of Money 
      Underlying ideas of Time Value of Money        13 
    General‐Problem Solving Techniques        13 
    Compound Interest              13 
      The Compound Amount Formula          14 
      Fractional Part of Compounding Periods        15 
      Manipulating the Compound Amount Formula      16 
      Effective and Nominal Interest Rates        17 

    Exercises                18 

CHAPTER 3. PRESENT VALUE 

      Present Value              20 
      Fractional Part of Compounding Periods        20 
      Equivalent Values              21 
    Exercises                22 

Workbook in Math of Investments    vi 
 
 

 
CHAPTER 4. SIMPLE ORDINARY ANNUITY 

    Annuities                23 
         Classification of Annuities            23 
    Ordinary Annuity              24
        Future Value of an Ordinary Annuity        24 
        Future Value using the Table          24 
        Present Value of an Ordinary Annuity        24 
        Present Value using the Table          24 
        Finding the Size of Each Periodic Payment       25 
        Present Value is known            25 
        Future Value is known            25
        Finding the Interest Rate per Period and Nominal Interest Rate   26  
        Present Value is known                       26 
        Finding the Term              27 
                   Future Value is known            27 
     Exercises                29 

 
CHAPTER 5. SIMPLE ANNUITY DUE 

      Future Value using the Formula                     31 
      Future Value using the Table                     31 
               Future Value using the Formula                     31 
      Future Value using the Table                     32 
    Additional Problems 
      Finding the Annuity Payment when Present Value is known           32 
      Finding the Annuity Payment when Future Value is known             32 
      Finding the Interest Rate per Period                    33 
      Finding the Term                         34 
    Exercises                           35 

 
CHAPTER 6. SIMPLE DEFERRED ANNUITY AND SIMPLE PERPETUITY 

     Deferred Annuity              37 
      Future Value using the Table          37 
      Present Value using the Table          37 
     Additional Problems 
      Finding the Annuity Payment when Present Value is known           38 

Workbook in Math of Investments    vii 
 
 

 
      Finding the Term when Present Value is known                                 38 
     Perpetuity                           39 
      Simple Ordinary Perpetuity                       39 
      Simple Perpetuity Due                       40 
                         Exercises                           41 
 
CHAPTER 7. SINKING FUND AND AMORTIZATION 

     Sinking Fund                43 
      Sinking Fund Payment using the Table        43 
      Sinking Fund Payment using the Formula        43 
      Sinking Fund Schedule            44 
  Amortization                           45 
      Amortization Payment using the Table        45 
      Amortization Payment using the Formula        46 
      All Periodic Payments are Equal          46 
     All Periodic Payments Except the Final Payment are Equal    47
   
                        Exercises                           48 

CHAPTER 8. DEPRECIATION 

     Definition of Terms               50 
   Methods Used for Financial Statement Reporting 
    Straight‐line Method                        50 
      Sum‐of‐the‐Years’ Digit Method          51 
      Declining‐Balance Method            52 
      Units‐of‐Production Method           53 
    Compound Interest Method             
      Annuity Method              54 
      Sinking Fund Method            55
   
                        Exercises                           56 
 
CHAPTER 9. STOCKS, BONDS AND MUTUAL FUNDS 

    STOCKS  
      Definition of Terms               58 
    Stock Quotation              69 
    Dividends on Preferred and Common Stock                               60 
Workbook in Math of Investments    viii 
 
 

 
      Current Yield for a Stock              62 
      Price‐Earnings Ratio of a Stock            62 
      Cost, Proceeds and Gain (Loss) on a Stock Transaction     62 
      Return on Investment             63 
    BONDS         
      Cost, Proceeds and Gain (Loss) on a Stock Transaction     63 
      Current Yield for a Bond             64 
    MUTUAL FUNDS                
      Definition and the Net Asset Value           64 
      Return on Investment             65
     
Exercises                            66 
 
APPENDIX – Present Value/Future Value Tables 

    Present Value Interest Factor          68 
    Present Value Interest Factor for an Annuity      70 
    Future Value Interest Factor            71 
    Future Value Interest Factor  for an Annuity       72 
     
 

Workbook in Math of Investments    ix 
 
 

 
Chapter 1 

SIMPLE INTEREST AND BANK DISCOUNT 
 
SIMPLE INTEREST 
 
Money 
 

 
Lender  Borrower 
  Money 

 
interest 
Lender/creditor – the one who invests the money 
Borrower/debtor ‐ the one who owes the money lent 
Interest – the income earned by the lender; the cost for the use of money by the borrower 
 
At the end of the time period, the borrower repays the amount originally owed plus the interest.  
 
Interest can be computed by two methods: 
1. Simple interest is an interest computed on the amount the borrower received at the time 
the Ioan is obtained and is added to that amount when the loan becomes due.  
‐Thus, simple interest is computed only once for the entire time period of the loan.  
‐Usually applied to loans whose time period is less than a year. 
2. Compound interest are the interest computed more than once during the time period of the 
loan.   
‐These are generally for time periods of a year or longer. 

FORMULAS  
Interest  

𝑰𝒏𝒕𝒆𝒓𝒆𝒔𝒕 𝑷𝒓𝒊𝒏𝒄𝒊𝒑𝒂𝒍 𝒙 𝑹𝒂𝒕𝒆 𝒙 𝑻𝒊𝒎𝒆 


Illustration: Find the interest to be paid by A for a loan of 200,000 for one year at a simple interest 
rate of 4%. 

Principal is the amount or deposit made by a depositor or the face amount lent to the borrower on 
loan date.   Ex. 200,000 
Simple interest rate (expressed in percentage) is converted to a decimal far computation purposes. 
Unless otherwise stated, the simple interest rate is an annual rate. Ex. 4% or 0.04 
Time is the length of time for which the money is borrowed or lent. The time expressed in years or 
fractional part of a year is the period between the loan date and maturity date. 

  date when the loan 
date when the loan was obtained  becomes due ex. 1 yr. 
 

Workbook in Math of Investments    1 
 
The simple interest may now be computed using the formula I = PRT. Substituting the given in the 
sample: 

Interest  =  Principal x Rate x Time 
Interest   =  200,000 x 0.04 x 1 
Interest   =  P8,000 
 
Maturity value 
Maturity value is the sum of the principal and interest. MV = P + I or  

𝑴𝒂𝒕𝒖𝒓𝒊𝒕𝒚 𝑽𝒂𝒍𝒖𝒆 𝑷 𝒙 𝟏 𝑹𝑻  
The maturity value may now be computed using the formula given above. Substituting the given in 
the sample: 

Maturity Value  =  Principal + interest 
Maturity Value  =  200,000 + 8,000 
Maturity Value  =  208,000 
  or   
Maturity Value  =  P x (1 + RT) 
Maturity Value  =  200,000 x [1 + 0.04(1)] 
Maturity Value  =  200,000 (1.04) 
Maturity Value  =  208,000 
 

Time 
‐ the  period  between  the  loan  date  and  the  maturity  date.  This  is  converted  to  decimal  to 
facilitate the computation, if practicable. (e. g. 1 ½ years = 1.5; 2 years and 3 months = 2.25; 
3 years and 9 months = 3.75 years; 1 year and 7 months = 1 ).  

  Loan date  TIME 
Maturity date 
 

NOTE:  Since  the  simple  interest  rate  is  given  as  an  annual  rate,  the  time  should  be  prorated 
accordingly in 12 months or 4 quarter or 2 semi‐annuals or 6 bimonthly payments.  

Instances of time: 

1. Maturity date is unknown  
If  the  time  T  is  given  in  months  and  only  the  loan  date  is  stated,  the  maturity  date  shall 
coincide with the loan date. Thus, a loan obtained on June 13, 2018 payable in 4 months will 
mature on October 13, 2018. 
2. Year is unknown 
If either the loan date and maturity date does not mention the year, it shall be assumed that 
these  dates  fall  on  the  same  year.  For  example,  a  loan  that  was  granted  on  February  14, 
2018 and to mature on September 20 would mature on September 20, 2018. 
3. T is stated as a certain number of days 
1. Exact interest method – uses 365 as denominator (366 days in a leap year) 
2. Ordinary interest method – uses 360 days 

Workbook in Math of Investments    2 
 
Note: Ordinary interest method yielded a higher interest than exact interest method. 

4. Only loan date and maturity date are given 
a. Actual time – counting every day excluding the loan date until the maturity date 
b. Approximate time – assuming that each month has 30 days 

Four  possible  combinations  (also  known  as  Banker’s  Rule)  to  compute  T  where  only  the 
loan date and maturity date are given: 
 

Illustration: Count the time and approximate time from April 8, 2018 to September 20, 2018. 

 
 
 
 
 
 
So, which of the Banker’s Rule has lower or higher yield of interest? 

 
MANIPULATING THE SIMPLE INTEREST FORMULA 
1. Principal is unknown (In pesos) 
𝑰𝒏𝒕𝒆𝒓𝒆𝒔𝒕
𝐏𝐫𝐢𝐧𝐜𝐢𝐩𝐚𝐥  
𝑹𝒂𝒕𝒆 𝒙 𝑻𝒊𝒎𝒆
Illustration: A bank loaned Anna money at 8% simple interest for 90 days. If the interest is P4, 000, 
find the principal amount borrowed. 

Principal  =  𝐼
 
𝑅𝑥𝑇

Principal   =  4,000
 
90
8%𝑥
360

Principal  =  4,000
 
0.02

Principal  =  P200,000 

2. Rate is unknown (In percentage) 
𝑰𝒏𝒕𝒆𝒓𝒆𝒔𝒕
𝐑𝐚𝐭𝐞  
𝑷𝒓𝒊𝒏𝒄𝒊𝒑𝒂𝒍 𝒙 𝑻𝒊𝒎𝒆

Workbook in Math of Investments    3 
 
Illustration: If Anna applies for a P175, 000 loan in  a bank  the  interest of  which is P5, 810 for 125 
days. What interest rate is being charged?  

Rate  =  𝐼
 
𝑃𝑥𝑇

Rate  =  5,810
 
125
175,000 𝑥
360

Rate  =  5,810
 
60,763.8888

Rate  =  0.095616 

Rate    =  9.56% 

3. Time is unknown (In years, months or days) 
𝑰𝒏𝒕𝒆𝒓𝒆𝒔𝒕
𝐓𝐢𝐦𝐞  
𝑷𝒓𝒊𝒏𝒄𝒊𝒑𝒂𝒍 𝒙 𝑹𝒂𝒕𝒆
Illustration: What would be the time period of Anna’s loan for P266, 000, at 11% ordinary interest, if 
the amount of interest is P10, 150? 

Time  =  𝐼
 
𝑃𝑥𝑅

Time  =  10,150
 
266,000 𝑥 0.11

Time  =  10,150
 
29, 260

Time  =  .3468899 year x 360 

Rate    =  124.8 or 125 days 

   

Workbook in Math of Investments    4 
 
BANK DISCOUNT  
 
Proceeds 
 

 
Lender  Borrower 
 
Maturity 
 
value 
 

Bank discounts are interest deducted in advance.  
A bank discount is an interest computed on the maturity value of the loan and is deducted from 
that amount at loan date to determine the net amount to be received by the borrower.   
‐The amount of loan applied for at loan date is the maturity value of the loan.  
 
The bank discount is deducted from that amount to arrive at the proceeds‐the amount the borrower 
is  to  receive.    This  is  in  contrast  to  simple  interest,  which  is  altogether  paid  with  the  principal  at 
maturity date.  
 In  computing  for  the  bank  discount,  three  factors  are  being  considered:  maturity  value,  bank 
discount rate and time.  In this section, bank discount rate is referred to as discount rate or rate and 
bank discount as discount. 
 

Formulas  
𝑩𝒐𝒏𝒅 𝒅𝒊𝒔𝒄𝒐𝒖𝒏𝒕 𝑴𝒂𝒕𝒖𝒓𝒊𝒕𝒚 𝒗𝒂𝒍𝒖𝒆 𝒙 𝑫𝒊𝒔𝒄𝒐𝒖𝒏𝒕 𝑹𝒂𝒕𝒆 𝒙 𝑻𝒊𝒎𝒆 
Illustration: Allen availed of a P245, 000 loan at 14% discount rate for 9 months. Find the bank 
discount and proceeds of the loan.  

Maturity value – the amount applied for by the borrower on loan date. Ex. 245,000 
Bank discount rate – expressed as a percentage, is converted to decimal and is an annual rate. Ex. 
0.14 
Time – expressed in years or fractional part of a year, is a period between the loan and maturity 
date. Ex. 9 months or 9/12 or 0.75. 
 

The bank discount may now be computed using the formula BD = MV x R x T. Substituting the given 
in the sample: 

Bank discount  =  Maturity value x Discount Rate x Time 
Bank discount  =  245,000 x .14 x .75 
Bank discount  =  P25,725 
 
Proceeds 
Proceeds is the amount the borrower is to receive. P = MV ‐ BD or  

𝑷𝒓𝒐𝒄𝒆𝒆𝒅𝒔 𝑴𝑽 𝒙 𝟏 𝑹𝑻  

Workbook in Math of Investments    5 
 
The  proceeds  may  now  be  computed  using  the  formula  given  above.  Substituting  the  given  in  the 
sample: 

Proceeds   =  Maturity value – bank discount 
Proceeds   =  245,000 – 25,725 
Proceeds   =  219,275 
  or   
Proceeds   =  MV x (1 ‐ RT) 
Proceeds   =  245,000 x [1 ‐ 0.14(.75)] 
Proceeds   =  245,000 (1.105) 
Proceeds   =  219, 275 
 

PROMISSORY NOTES 
‐ A note evidencing indebtedness and commitment to pay. 
‐ an unconditional promise in writing made by one person to another, signed by the maker 
engaging to pay on demand or at a fixed or determinable future time a sum certain in money 
to order or to bearer (Negotiable Instruments Law). 
‐ A document stating the details of a loan is a negotiable instrument which, when properly 
endorsed, can be transferred or sold to another person or a bank which is nor a party to the 
original loan.   
 
Two parties involved: 
Maker – who makes the promise and who signs the instrument  
Payee – to whom the promise is made and to whom the instrument is payable 
 

Parts of the promissory note:  
1. Bank discount Note  
 
Face   
Value (FV)   
225, 000   Agdao, Davao City   June 13, 2018  Issue 
 
    Date 
Term   
75  days  after  date,  I  promised  to  pay  to  the  order  of  Upside  Payee 
 
Down, two hundred twenty five thousand and xx/100 pesos for 
 
the value received with bank discount at 8% per annum.   Bank 
 
discount 
1.     rate 
2.  
Maturity  3.   Due Aug 27, 2018      Thessa Sales 
date  Maker 
4.  
 
 
 
 
 
 
 

Workbook in Math of Investments    6 
 
 
2. Simple interest note  

  Face 
  Value (FV)  175, 000   Kabacan, Cotabato  May 13, 2018  Issue 
  Date 
 
Term  90  days  after  date,  I  promised  to  pay  to  the  order  of  Amplayo  Payee 
 
Grocery  store,  one  hundred  seventy  five  thousand  and  xx/100 
  pesos for the value received with an interest at 12% per annum.   Simple 
  interest 
 
rate 
  Maturity  Due Aug 11, 2018      Thalia Domingo 
  date  Maker 

Types of Notes: (interest‐bearing or non‐interest‐bearing) 

1. Simple interest note (FV = Principal) and (MV > FV)    Answer: Maturity Value = P180, 250 
2. The bank discount note (FV = MV) and (FV > Proceeds)  Answer: Proceeds   = P221, 250 
 

EFFECTIVE RATE OF A BANK DISCOUNT NOTE 
In a simple interest note, the borrower receives the full face value, whereas with a bank discount 
note the borrower receives only the proceeds. Because proceeds is less than the face value, the 
stated discount rate is not the true or effective rate of the note.  

Formula 
𝑩𝒂𝒏𝒌 𝒅𝒊𝒔𝒄𝒐𝒖𝒏𝒕 𝑩𝑫
𝑬𝒇𝒇𝒆𝒄𝒕𝒊𝒗𝒆 𝒊𝒏𝒕𝒆𝒓𝒆𝒔𝒕 𝒓𝒂𝒕𝒆  
𝑷𝒓𝒐𝒄𝒆𝒆𝒅𝒔 𝑷 𝒙 𝑻𝒊𝒎𝒆 𝑻
  Where: 
    BD = FV x DR x T  
    P = FV ‐ BD 
Illustration:  What  is  the  effective  interest  rate  of  a  bank  discount  note  for  P350,  000,  at  a  bank 
discount rate of 14% for a period of 6 months? To find the effective interest rate, the bank discount 
and proceeds should be computed first. The following procedures are observed: 

1. Solve for the bank discount 

Bank discount  =  Maturity value x Discount Rate x Time 
Bank discount  =  350,000 x .14 x .5 
Bank discount  =  P24,500 
2. Solve for the proceeds 

Proceeds  =  Maturity value – bank discount 
Proceeds  =  350,000 – 24,500 
Proceeds  =  325, 500 
 

Workbook in Math of Investments    7 
 
3. Solve for the effective interest rate 

Effective interest rate  =  𝐵𝑎𝑛𝑘 𝑑𝑖𝑠𝑐𝑜𝑢𝑛𝑡


 
𝑃𝑟𝑜𝑐𝑒𝑒𝑑𝑠 𝑥 𝑇𝑖𝑚𝑒

Effective interest rate  =  24,500
 
325,500 𝑥 .5

Effective interest rate  =  24,500
 
162,750

Effective interest rate  =  .15053 or 15.05 % 

 
 
 
 
 
 
 
 
 
 
 
 
 
 
 
 
 
 
 
 
 

Workbook in Math of Investments    8 
 
DISCOUNTING 
Discounting Notes before Maturity 
For a company or individual to cash in a note at any time before maturity, the payee (lender) 
may take the note to a bank and sell it. This process is known as discounting a note. 

In discounting a note, the original payee receives the proceeds of the discounted note, and 
the bank (the new payee) receives the maturity value of the note when it matures. The time used to 
compute  the  proceeds  is  from  the  date  the  note  is  discounted  to  the  maturity  date  known  as  the 
discount period. 

 
Discounting a Simple Interest Note 
Illustration: XYZ Distributors received a P150, 000 simple interest note for 5 months at 12% simple 
interest from one of its customers. After 3 months, XYZ distributors needed cash so it discounted the 
note  at  the  ABC  bank  at  a  discount  rate  of  14%.  What  are  the  proceeds  XYZ  will  receive  from  the 
discounted note? 

The following procedures are observed for simple interest note: 

1. Solve for the maturity value of the original note.  

Maturity Value  =  Principal (1 + RT) 
Maturity Value  =  150,000 [1 + (.12 x 5/12)] 
Maturity Value  =  150,000 (1 + .05) 
Maturity Value  =  P 157, 500 
2. Count the number of months or days of the discount period: In this illustration, the discount 
period is 2 months – 5 months less the 3 months that had lapsed. 
3. Solve for the bank discount.  

Bank discount  =  Maturity value x Discount rate x Time 
Bank discount  =  157, 500 x .14 x 2/12 
Bank discount   =  P 3, 675 
 
4. Solve for the proceeds.  

Proceeds  =  Maturity value – bank discount 
Proceeds  =  157,500 – 3,675 
Proceeds  =  P 153, 825 
 

Discounting a Bond Discount Note 
Illustration: XYZ Distributors received a P350, 000 bank discount note for 6 months from one of its 
customers. After 3 months, XYZ distributors discounted the note at the ABC bank at a discount rate 
of 14%. What are the proceeds XYZ will receive from the discounted note? 

The  following  procedures  are  observed  for  bank  discount  note:  (same  with  simple  interest  note 
except for step 1: No need to compute for maturity value because this is the face value) 

Workbook in Math of Investments    9 
 
1. Count the number of months or days of the discount period. In this illustration, the discount 
period is 3 months – 6 months less the 3 months that had lapsed 
2. Solve for the bank discount.  

Bank discount  =  Maturity value x Discount rate x Time 
Bank discount  =  350,000 x .14 x 3/12 
Bank discount   =  P 12, 250 
 
3. Solve for the proceeds.  

Proceeds  =  Maturity value – bank discount 
Proceeds  =  350, 000 – 12,250 
Proceeds  =  P 337, 750 
 

   

Workbook in Math of Investments    10 
 
CHAPTER 1 EXERCISES 
I. SIMPLE INTEREST 
1. Find the simple interest on P8, 000 at an annual interest rate of 12% for two years. 
2. Find the simple interest on a P30, 000 loan due in 5 years when the annual interest rate 
on the loan is 16%. What is the maturity value of this loan? 
3. Find the simple interest on a P50, 000 loan at 14 ½ % for 7 months. 
 
II. CONCEPT OF TIME 
a. Find the time, in days, of each of the following notes using approximate and actual time: 
1. January 10, 2000 to February 18, 2000           
   
2. March 6, 2000 to November 15, 2000           
   
3. May 30, 2001 to August 16, 2013             
   
 
b. Find the due dates of each of the following notes: 
Date of the loan    Time          Due Date 
1. April 10, 2011      3 months          
   
2. August 18, 2014      6 months         
   
3. July 31, 2019      90 days          
   
 
c. Find the Exact Interest and Ordinary Interest 
  Principal  Rate   Time  Exact  Ordinary 
Interest  Interest 
1.  450, 000  13%  100 days         
2.  9, 000  10 ¼ %  60 days         
3.  504, 700  9 1/8 %  58 days         
 
III. MATURITY VALUE  
Principal     Rate      Time 
1. 540,000    11.9%      2 years         
2. 1,250,000    12 ½ %      5 months       
3. 750,000    14%      6 months        
4. 186,200    10 ½ %      30 months       
5. 7,500,000    13.35%     11 months        
 
IV. MANIPULATING THE FORMULAS 
  Principal  Rate  Time  Interest  Interest  Maturity Value 
Method 
1.        9.5%  100 days  Exact  3,400       
2.  150,000  14%      Ordinary  9,600       

Workbook in Math of Investments    11 
 
3.  36,000      160 days  Exact  2,250       
 

V. BANK DISCOUNT 
  Face Value  Discount  Date Of  Term  Maturity  Bank  Proceeds 
Rate  Note  Date  Discount 
1.  50,000  14.7%  Apr 16      July 9         
2.  8,750,000  9 ½ %  Oct 25  87 days             
3.  8,000  12.1%  Sep 3  109 days             
 
VI. PROMISSORY NOTE (1‐3 SIMPLE INTEREST NOTE AND 4‐6 BANK DISCOUNT 
NOTE) 
  Face  Int.  Note  Term   Maturity  Maturity  Date of  Disc.  Disc.  Proceeds 
value  rate  date  date  value  disc.  period  Rate 
1.  40,000  10.4%  Dec 12  50 days          Jan 19      15%     
2.  55,000  12%  Mar 4  70 days          Apr 15      13%     
3.  15, 000  15%  May 13  80 days          July 10      17%     
4.  80,000  11.4%  Jan 13  60 days          Feb 20      16%     
5.  50,000  13%  Apr 5  80 days          May 16      14%     
6.  30,000  16%  Feb 27  68 days          Apr 7      18 %     
 
VII. WORD PROBLEMS 
1. Find the (a) exact and (b) ordinary simple interest on a 120‐day loan of P1, 450,000 that 
has an annual interest rate of 19 ¼ %. (c) Which gives the lender a greater return on the 
investment and by how much? 

2. Find the exact time from Feb 4 to Apr 21 of the year 2016. Then, using the Banker’s rule, 
find the interest on P18, 000 at 17 1/6 %. 
 

3. Alex borrowed money at 9% interest for 125 days. If the interest charge was P56, 000, 
use the ordinary interest method to compute the amount of principal of the loan. 

4. Alison goes to the bank and borrows P150, 000 at 9 ½ % for 250 days. If the bank uses 
the ordinary interest method, how much will Alison have to pay? 
 

5. Angelica  signed  a  P240,  000  bank  discount  note  at  the  CocoBank.  The  discount  rate  is 
14%,  and  the  note  was  made  on  February  19,  for  50  days.  (a)  What  proceeds  will 
Angelica receive on the note and its maturity value? 

Workbook in Math of Investments    12 
 
Chapter 2 

COMPOUND AMOUNT OR FUTURE VALUE 
Time value of money – a peso received today is worth more than a peso received tomorrow. 
The difference between a present value and a future value is the interest that is included in the 
future amount. Interest accrues over time. 
 
PV – FV = 2 factors: 
1. Interest rate. The greater the rate, the larger the interest, so is Future Value. 
2. Length of time. The longer the time, the larger the interest, so is Future Value. 

Underlying ideas of time value of money: 
 A present value is always less than a future amount 
 A future amount is always greater than a present value 
 A  peso  available  today  is  always  worth  more  than  a  peso  that  does  not  become  available 
until a future date 
 A peso available at a future date is always worth less than a peso that is available today 

GENERAL PROBLEM‐SOLVING TECHNIQUES   
The following steps will help categorize which investment math problem is at hand.   
1.  Determine  if  the  problem  involves  a  single  payment  or  a  sequence  of  equal  periodic  payments.  
Simple  and  compound  interest  problems  involve  a  single  present  value  and  a  single  future  value.  
Ordinary  annuities  may  be  concerned  with  a  present  value  or  a  future  value  but  always  involve  a 
sequence of equal periodic payments.   
2. If a single payment is involved, determine if simple or compound interest is used.  Simple interest 
is usually used for durations of a year or less and compound interest for longer periods.    
3. If a sequence of periodic payments is involved, determine if the payments are being made into an 
account that is increasing in value‐a future value problem‐or if payments are being made out of an 
account that is decreasing in value‐a present value problem.  
 
 
COMPOUND INTEREST 
 
Compound interest – in contrast to simple interest. Computes interest more than once during the 
term of the loan or investment. It yields higher interest than simple interest because the interest are 
earning interest. The relationships between the present value and the future amounts assume that 
the interest earned on the investment is reinvested, rather than withdrawn. Reinvesting the interest 
causes the investment to increase, thus causes more interest to be earned in each successive period.  

𝑪𝒐𝒎𝒑𝒐𝒖𝒏𝒅 𝑰𝒏𝒕𝒆𝒓𝒆𝒔𝒕 𝑭𝒖𝒕𝒖𝒓𝒆 𝑽𝒂𝒍𝒖𝒆 𝒍𝒆𝒔𝒔 𝑷𝒓𝒆𝒔𝒆𝒏𝒕 𝑽𝒂𝒍𝒖𝒆 


Compound  amount  –  the  final  sum  of  principal  and  accumulated  interest  at  the  end  of  the 
borrowing period 
Compounding  period  –  the  period  for  computing  the  interest  which  has  regular  intervals  such  as 
annually, semi‐annually, quarterly, or monthly 
Compounding – the process of finding a compound amount when the present value is known 

Workbook in Math of Investments    13 
 
 

The Compound Amount Formula 
𝑨 𝑷 𝟏 𝒊 𝒏 
Where: 

A = compound amount of future value 
P = present value or principal 
i = interest rate per period, expressed as a decimal 
n = total compounding periods 
 

In  practice,  there  are  two  methods  for  solving  compound  interest  problems.  The  first  uses  the 
compound amount table (see annex Table 1) and the second uses the compound amount formula.  

Using the Table 

𝑪𝒐𝒎𝒑𝒐𝒖𝒏𝒅 𝑽𝒂𝒍𝒖𝒆 𝑭𝑽 𝑷𝒓𝒊𝒏𝒄𝒊𝒑𝒂𝒍 𝒙 𝑻𝒂𝒃𝒍𝒆 𝑭𝒂𝒄𝒕𝒐𝒓 


In order to use the compound amount table, the number of compounding periods and the interest 
rate per period must be known. 

The chart below serves as a guide in finding the number of compounding periods per year. 

Interest  Compounding made  Compounding 


compounded  periods per year 
Annually  Every year  1 
Semi‐annually   Every 6 months   2 
Quarterly  Every 3 months  4 
Monthly   Every months  12 
Daily   Every day  365 
 

The chart below is a useful tool in determining the interest rate per period. 

Nominal  Interest  Compounding  Interest rate 


interest rate  compounded  periods per  per period 
year  (Nominal rate 
/ periods per 
year) 
9%  Annually  1  9% 
12%  Semi‐annually  2  6% 
16%  Quarterly  4  4% 
18%  Monthly  12  1.5% 
21%  Quarterly  4  5.25% 
 

Illustration:  Bea  invested  P12,  000,  at  8%  interest  compounded  quarterly,  for  5  years.  Use  the 
compound amount table or Table 1 to find the compound amount of Bea’s investment. 

Interest rate per period  =  8% ÷ 4 = 2% 

Workbook in Math of Investments    14 
 
Compound periods  =  5 x 4 = 20 

Compound amount   =  Principal x Table factor  
Compound amount  =  P12, 000 x 1.485947 (Table 1) 
Compound amount   =  P17, 831.40 

Compound interest  =  Compound amount – Principal 
Compound interest  =  17, 831.40 – 12, 000 
Compound interest  =  P5, 831.40 
 

Note:  For  the  periods  beyond  the  table,  a  new  table  factor  can  be  provided  by  multiplying  the 
factors for any two periods that add up to the number of periods required. (e. g. Table factor for 72 
periods  would  be  the  product  of  the  factors  for  40  and  30  periods,  35  and  37,  or  any  other 
combination that adds up to 72.) Same rule will apply for the table factors for present value to be 
discussed in next chapter. 

 
Steps for Fractional Part of Compounding Periods 
Illustration: Find the compound amount and the compound interest when P10, 000 is invested for 3 
years and 2 months at 6% compounded semi‐annually, the procedures will be used: 

1. Find the compound amount for the whole compounding periods. (A = P X Table factor) 

Compound amount   =  Principal x Table factor  
Compound amount  =  P10, 000 x 1.194052 
Compound amount   =  P11, 940.52 

2. Solve  for  the  simple  interest  for  the  remaining  fractional  period  based  on  the  compound 
amount on step 1. (SI = A X R X T) 

Simple interest  =  Principal x Rate x Time 
Simple interest  =  11, 940.52 x .06 x 2/12 
Compound amount   =  P119.40 

3. Compute for the final compound amount by adding the A and SI. 

Final compound amount  =  Compound  amount  +  simple 


interest 
Final Compound amount  =  11, 940.52 + 119.40  
Final Compound amount   =  P12,059.92 

The compound interest is found by subtracting the principal from the final compound amount. 

Compound interest  =  Final compound amount – Principal 
Compound interest  =  12, 059.92 – 10,000  
Compound amount   =  P2, 059.92 

   

Workbook in Math of Investments    15 
 
 

Manipulating the Compound Amount Formula 
1. Nominal interest rate or time is unknown 

𝒏
𝐴
𝟏 𝒊  
𝑃
In finding the value of i, sometimes the factor cannot be located. It’s between two rates. To get the 
exact interest rate, interpolation method can be used.  
 
Illustration:  At  what  nominal  interest  rate  compounded  semi‐annually  for  10  years  will  P30,  000 
accumulate to P89, 000? 
 
𝟏 𝒊 𝒏
  =  𝐴
 
𝑃
1 𝑖   =  89,000
 
30,000
1 𝑖   =  2.966667 

Exponent n is equal to 20 (2 compounding periods per year x 10 years). The factor cannot be located 
in the table. It is in between of 5 ½% and 6%.  
 
  Interest period per  Table factor    
period 
  6%  3.207135  (1) 

  i   2.966667  (2) 

  5 1/2%  2.917757  (3) 

(2)‐(3)  x – 5 ½ %  0.048910  (4) 



(1)‐(3)      ½ %  0.289378  (5) 
 
Solve for i from the proportion formed by the differences on lines (4) and (5) 

=  1 0.048910 
i  – 5 ½ %  % 
2 0.289378 
i  – 5 ½ %  =  0.000845 
i    =  0.000875 + .055 
i    =  0. 055845 
The desired value of i is .055845 or 5.58 % per semi‐annual period. So, the nominal rate per annum is 
11.17%. 

Interpolation method can also be used for the value of T if the table factor cannot be found.  

Workbook in Math of Investments    16 
 
Effective and Nominal Interest Rate 
 
Nominal rate – the advertised or stated interest rate 
Effective rate –the real rate of return of an investment 
 
𝐓𝐨𝐭𝐚𝐥 𝐜𝐨𝐦𝐩𝐨𝐮𝐧𝐝 𝐢𝐧𝐭𝐞𝐫𝐞𝐬𝐭 𝐞𝐚𝐫𝐧𝐞𝐝 𝐢𝐧 𝐨𝐧𝐞 𝐲𝐞𝐚𝐫
𝑬𝒇𝒇𝒆𝒄𝒕𝒊𝒗𝒆 𝑰𝒏𝒕𝒆𝒓𝒆𝒔𝒕 𝑹𝒂𝒕𝒆  
𝑷𝒓𝒊𝒏𝒄𝒊𝒑𝒂𝒍
or  

𝒊𝑵𝒐𝒎 𝒎
𝒊𝑬𝒇𝒇 𝟏 𝟏 
𝒎
Where: 

    𝑖 = nominal rate 
    𝑚 = number of compounding periods for one year 
    i = interest rate per period (𝑖 𝑚 
    𝑖 = effective rate 
   

Workbook in Math of Investments    17 
 
CHAPTER 2 EXERCISES 
 
I. COMPOUNDING AMOUNT  
a. Convert the Nominal Interest Rate to the Interest Rate per Period and Determine the Total 
Number of Interest Periods: 

  Nominal  Term Of Loan  Interest Compounded  Periodic Rate  Number Of 


Rate  Periods 
1.  14%  4 years  Annually         
2.  12%  8 years  Quarterly         
3.  13%  3 years  Monthly         
4.  12%  9 months  Quarterly         
5.  14%  4 years  Semi‐annually         
 
b. Find The Compound Amount And Compound Interest 

  Principal  Term Of  Nominal  Interest Compounded  Compound  Compound 


Investment  Rate  Amount  Interest 
1.  P 50,000  1 year  12%  Quarterly         
2.  80,000  3 years  8%  Semi‐annually         
3.   400,000  1 ½ years  8%  Semi‐ annually         
4.  142,500  10 years  16%  Quarterly         
5.  60,000  2 years  24%  Bi‐monthly         
 
c. Creating a New Table Factor and Finding the Compound Amount 

  Principal  Term Of  Nominal  Interest Compounded  New Table  Compound 


Investment  Rate  Factor  Interest 
1.  190,000  60 years  9%  Annually         
2.  200,000  13 years  16%  Semi‐annually         
3.   347, 000  20 years  16%  Quarterly         
 
d. Find the Compound Amount with Fractional Part of Compounding Periods 

  Principal  Interest  Interest Compounded  Time  Compound 


Rate  Amount 
1.  4,000  18%  Monthly  10 ½ months       
2.  17,000  8%  Quarterly  4 years, 2 months       
3.  36,000  10%  Annually  20 years, 5 months       
 
e. Find the missing figure of interest rate and number of compounding periods 

  Principal  Future  Interest Compounded  Number Of  Interest Rate 


Value  Compounding  Per Period 
Periods 
1.  50,000  114,860  Monthly  3 ½ years        
2.  25,000  29,600  Quarterly  4 years       
3.  400,000  916,800  Quarterly  4 ¼ years       
4.  30,000  46,890  Monthly        18% 
5.  20,000  32,380  Quarterly        14% 
6.  50,000  87,050  Quarterly        8% 

Workbook in Math of Investments    18 
 
II. EFFECTIVE INTEREST 
  Principal  Nominal  Interest Compounded  Compound Interest  Effective 
Rate  In One Year  Interest Rate 
1.  50,000  10%  Semi‐annually             
2.  85,000  12%  Monthly             
3.  10,000  8%  Quarterly             
4.  30,000  46,890  Monthly             
 

III. WORD PROBLEM 
1. (a)  Find  the  simple  interest  on  P60,  000  for  one  year  at  10%.  (b)  Find  the  compound 
interest  if  the  same  investment  is  compounded  semi‐annually?  (c)  How  much  is  the 
compound interest greater than the simple interest? 

2. Ace deposited P500, 000 in an account earning 12% compounded monthly. This account 
is intended to pay for the construction of a new warehouse. How much will be available 
for the project in 2 ½ years? 
 

3. Accumulate  P300,  000  for  eight  years  at  7%  compounded  quarterly.  How  much  is  the 
interest? 

4. Find the difference between two investments: (a) 150,000 is invested for the years at 8% 
compounded monthly; and (b) 150,000 is invested for five years at 8% simple interest. 

 
5. On  Jan  1,  1998,  Abram  borrowed  P12,  000  and  agreed  to  repay  it  with  P4,  650.70 
interest. If the interest is at 6% compounded quarterly, what amount must repay and on 
what date? 
 
6. Agatha invested P30, 000 at the Prime Bank, at 6% interest compounded quarterly. (a) 
What is the effective interest rate of this investment? (b) What will Agatha’s investment 
be worth after 6 years? 

   

Workbook in Math of Investments    19 
 
Chapter 3 

PRESENT VALUE 
PRESENT VALUE – the amount that must be invested today in order to accumulate with compound 
interest  to  the  future  value  given.  This  value  is  crucial  because  it  provides  financial  people  with  a 
basis for comparing the profitability of different projects or investments over a period of time. This is 
the cash value of future returns or income once a discount rate has been applied to it. 

 
𝒏
𝑷𝑽 𝑨 𝟏 𝒊  
Where: 
PV = present value 
A = compound amount 
i = interest rate per period, expressed in decimal 
n = total compounding periods 
 
The PV factors are less than one, because the original investment is less than the compound amount. 
(unlike the table factors of compound interest which are greater than one) 
 

Fractional Part of Compounding Periods 
There are cases when the time are with a fractional part. The steps to compute for the final present 
value are as follows:  

Illustration:  Find  the  present  value  of  a  non‐interest  bearing  note  of  P10,  000  for  three  years  and 
two months at 6% compounded semiannually.  

1. Find the present value for the whole compounding periods plus 1 period. Refer to Table 2. 
PV = Compound amount x Table Factor 
PV = 10,000 x 0.813092 = P8, 130.92 
 
2. Compute simple interest for the difference between the period in step 1 and the subject period. 
SI = PRT = 8,130.92 x .06 x 4/12 = P162.62 
 
3. Add the simple interest to the resulting value in step 1 to arrive at the present value. 
Final PV = PV + SI 
Final PV = 8,130.92 + 162.62 = P8, 293.54 
 
4. Additionally, compound amount less final present value equals compound interest. 
CI = Compound amount – Final PV 
CI = 10,000 – 8,293.54 = P1, 704.46 
 

   

Workbook in Math of Investments    20 
 
Equivalent Values 
As long as interest is involved, a sum of money can have different values at a different times. A single 
or  set  of  obligations  may  be  replaced  by  another  single  or  set  of  obligations  set  on  different  due 
date. Let’s look at the equation of value that makes the original obligations and the new obligations 
be equal value on a comparison date.  

Illustration:  Jude  owes  P30,  000  due  in  three  years  and  P40,  000  due  in  eight  years.  He  and  his 
creditor have agreed to settle the debts by two equal payments in five and six years, respectively. 
Find  the  size  of  each  payment  if  money  is  worth  6%  compounded  semiannually.  Let  x  be  each 
payment and the comparison date be the six years from now. The values on the comparison date are 
computed as follows: 

1. The value of the old debt of 30,000 becomes P35, 821.56 on the comparison date. Consider 
PV  =  30,000,  i  =  3%  (6%  /  2),  n  =  6  (3  years  –  from  due  date  to  comparison  date  x  2 
compounding periods per year). The compound amount table or Table 1 is used. 

Compound amount  =  Present value x Table factor 
Compound amount  =  30,000 x 1.194052 = P35,821.56 
 

2. The value of the old debt of 40,000 becomes P35, 539.48 on the comparison date. Consider 
A  =  P40,  000,  i  =  3%,  n  =  4  (2  years  from  the  comparison  date  to  the  due  date  x  2 
compounding periods per year). The present value table or Table 2 is used. 

Present value  =  Compound amount x Table factor 
Present value  =  40,000 x 0.888487= P35,539.48 
 

3. The value of the new debt, which is the first payment due in 5 years, becomes 1.0609x on 
the comparison date. Consider PV = x, i = 3%, n = 2 (1 year from the 5th year – the date on 
which  the  first  equal  installment  is  made  to  the  6th  year  –  the  comparison  date  x  2 
compounding periods per year). Refer to Table 1. 

Compound amount  =  Present value x Table factor 
Compound amount  =  X (1.0609) = 1.0609x 
 

4. The  value  of  the  second  payment  due  in  six  years  is  x.  it  does  not  change  since  the 
comparison  date  is  also  six  years.  The  equation  of  value  based  on  the  comparison  date  is 
given below: 

New debts  =  Old debts 
x + 1.0609x  =  35,821.56 + 35,539.48 
x + 1.0609x  =  71,361.04 
2.0609x  =  71,361.04 
X  =  34,626.15 
 

Hence, Jude should pay 34,626.15 at the end of the 5th and 6th year, respectively. 

   

Workbook in Math of Investments    21 
 
CHAPTER 3 EXERCISES 
I. PRESENT VALUE  
a. Find the Present Value and The Compound Interest Using Table and Using Formula 

  Compound Amount  Term Of  Nominal  Interest  Present  Compound 


Investment   Rate  Compounded  Value  Interest 
1.  60,000  3 years  9%  Annually         
2.  500,000  25 years  10.50%  Annually         
3.  2,500  1 year  18%  Monthly         
4.  9,000  4 years  9%  Semi‐annually         
5.  100,000  2 years  12%  Bi‐monthly         
 
b. Creating a New Table Factor and Finding the Present Value  

  Principal  Term Of  Nominal  Interest  New Factor  Present 


Investment   Rate  Compounded  Value 
1.  120,000  30 years  16%  Quarterly         
2.  1,100,000  55 years  8%  Semi‐annually         
3.  330,000  52 years  7%  Annually         
 

II. WORD PROBLEMS 
1. Angelo wants to renovate his house in 3 years. He estimates the cost to be P300, 000. 
How  much  must  Angelo  invest  now  at  8%  interest  compounded  quarterly,  in  order  to 
have P300, 000, 3 years from now? 

2. Anne is planning a vacation in Europe in 4 years, after graduation. She estimates that she 
will need P350, 000 for the trip. (a) If her bank is offering 4‐year certificates of deposit 
with  8%  interest  compounded  quarterly,  how  much  must  Anne  invest  now  in  order  to 
have  the  money  for  the  trip?  (b)  How  much  compound  interest  will  be  earned  on  the 
investment? 
 

3. If  P360,  000  is  due  in  seven  years  from  now  and  money  is  worth  5%  compounded 
annually, find the present value and the compound interest. 

4. What principal will accumulate to P320, 000 in 4 years at 4% compounded quarterly? 
 

5. Find the present value of P140, 000 due at the end of nine years if money is worth (a) 5% 
compounded  quarterly  and  (b)  7%  compounded  semiannually.  How  much  is  the 
compound interest in each case? 

   

Workbook in Math of Investments    22 
 
Chapter 4 

SIMPLE ORDINARY ANNUITY 
Annuities  –  series  of  equal  periodic  payments  or  receipts,  rather  than  lump  sums  which  is  the 
concern  of  compound  interest.  Originally,  this  only  refers  to  annual  equal  payments,  but  is  now 
applies to payment intervals of any length of time.  
The  subject  of  annuities  affects  business  firms  as  well  as  household,  such  as  periodic  savings,  life 
insurance premiums, interest payments on bonds and purchases of cars, houses, or home appliances 
on installment payment plans. 
 
Payment interval – the period of time between two successive payments dates  
 
Term  –  the  time  between  the  beginning  of  the  first  payment  interval  and  the  end  of  the  last 
payment interval 
 

 
Classifications of Annuities 
1. By term 
a) Annuity certain – term begins and  ends on definite dates(e.g.  5‐year term from Jan 1, 
2019 – Jan 1, 2024) 
b) Perpetuity – term begins on a definite date but never ends (e.g. drawing interest) 
c) Contingent annuity – term begins on a definite date, but the ending date is not fixed in 
advance which depends on some conditions happening in the future. (e.g. life insurance 
premium) 
2. By dates of payment 
a) Ordinary  annuity  –  periodic  payments  are  made  at  the  end  of  each  payment  interval 
(e.g. On Jan 1, 1 year interest payment is made quarterly. So, April 1 is the first payment) 
b) Annuity due  – periodic  payments are  made at the  beginning of  each payment  interval 
(e.g. 1 year interest payment quarterly starting Jan 1) 
c) Deferred annuity – same with ordinary except the term of annuity does not begin until 
after a designated period of time. (e.g. debtor with P20,000 loan on Jan 1, 2019 agrees 
to three equal payments starting 2022) 
3. By length of payment interval and interest compounding period 
a) Simple annuity – payment interval coincides with the interest compounding period. (e.g. 
payment interval is monthly, thus, the interest is compounded monthly) 
b) Complex  annuity  or  general  annuity  ‐  payment  interval  does  not  coincides  with  the 
interest  compounding  period.  (e.g.  payment  interval  is  monthly,  the  interest  is 
compounded quarterly) 

Workbook in Math of Investments    23 
 
 

ORDINARY ANNUITY 
Future Value of Ordinary Annuity 
Periodic payments are made at the end of each payment interval. 
𝒏
𝒏 𝟏 𝒊 𝟏
𝑭𝑽𝑶𝑨  
𝒊
  Where: 

    𝐹𝑉  = Future value of an ordinary annuity 
    Pmt = Annuity payment 
    i = interest rate per period (nominal rate ÷ periods per year) 
    n = Number of periods (years x periods per year) 
 
Using the Table 
Illustration: Bea deposited P30, 000 at the end of each year for 8 years in her savings account. If her 
bank paid 5% interest compounded annually, find the future value of Bea’s account. Use the Table 3 
for the future value of annuity.  

Future value   =   Annuity  payment  x  table 


factor 
Future value  =  30,000 x 9.549109 
Future value  =  P286, 473.27 
 

Present Value of Ordinary Annuity 
𝒏
𝟏 𝟏 𝒊
𝑷𝑽𝑶𝑨  
𝒊
  Where: 

    𝑃𝑉  = Present value of an ordinary annuity 
    Pmt = Annuity payment 
    i = interest rate per period (nominal rate ÷ periods per year) 
    n = Number of periods (years x periods per year) 
 
Using the Table 
Illustration:  How  much  must  be  deposited  now,  at  9%  interest  compounded  annually,  to  yield  an 
annuity payment of P50, 000 at the end of each year, for 10 years? Use the Table 4 for the present 
value of annuity.  

Present value   =   Annuity payment x table factor 


Present value  =  50,000 x 6.417658 
Present value  =  P320, 882.90 
 

Workbook in Math of Investments    24 
 
Finding the Size of Each Periodic Payment 
Present value is known 
To find the size of each payment, either of the formula may be used: 

𝐏𝐫𝐞𝐬𝐞𝐧𝐭 𝐕𝐚𝐥𝐮𝐞
𝑨𝒏𝒏𝒖𝒊𝒕𝒚 𝒑𝒂𝒚𝒎𝒆𝒏𝒕 𝑻𝒂𝒃𝒍𝒆 𝟒 
𝐓𝐚𝐛𝐥𝐞 𝐅𝐚𝐜𝐭𝐨𝐫
or 

𝑨𝒏𝒏𝒖𝒊𝒕𝒚 𝒑𝒂𝒚𝒎𝒆𝒏𝒕 𝐏𝐫𝐞𝐬𝐞𝐧𝐭 𝐕𝐚𝐥𝐮𝐞 𝐗 𝐓𝐚𝐛𝐥𝐞 𝐅𝐚𝐜𝐭𝐨𝐫 𝑻𝒂𝒃𝒍𝒆 𝟓 


 

Illustration: The present value of an annuity for ten years is P10, 000. Find the size of the quarterly 
payment if the interest rate is 8% compounded quarterly. 

Substitute the value of the following: PV = P10, 000; i = 2%; n = 40 

Annuity payment  =  Present Value


 
Table Factor
Annuity payment  =  10,000
 
27.355479
Annuity payment  =  365.56 
    Or 
Annuity payment  =  Present Value x Table Factor 
Annuity payment  =  10,000 x 0.036556 
Annuity payment  =  365.56 
 

Future value is known 
To find the size of each payment, either of the formula may be used: 

𝐅𝐮𝐭𝐮𝐫𝐞 𝐕𝐚𝐥𝐮𝐞
𝑨𝒏𝒏𝒖𝒊𝒕𝒚 𝒑𝒂𝒚𝒎𝒆𝒏𝒕 𝑻𝒂𝒃𝒍𝒆 𝟑 
𝐓𝐚𝐛𝐥𝐞 𝐅𝐚𝐜𝐭𝐨𝐫
or 

𝑨𝒏𝒏𝒖𝒊𝒕𝒚 𝒑𝒂𝒚𝒎𝒆𝒏𝒕 𝐅𝐮𝐭𝐮𝐫𝐞 𝐕𝐚𝐥𝐮𝐞 𝐗 𝐓𝐚𝐛𝐥𝐞 𝐅𝐚𝐜𝐭𝐨𝐫 𝐢 𝑻𝒂𝒃𝒍𝒆 𝟓 


 

Illustration: The future value of an annuity for ten years is P10, 000. Find the size of the quarterly 
payment if the interest rate is 8% compounded quarterly. 

Substitute the value of the following: FV = P10, 000; i = 2%; n = 40 

Workbook in Math of Investments    25 
 
Annuity payment  =  Future Value
 
Table Factor
Annuity payment  =  10,000
 
60.401983
Annuity payment  =  165.56 
  or   
Annuity payment  =  (Future Value x Table Factor 𝑖 ) 
Annuity payment  =  10,000 x (0.036556 – 0.02) 
Annuity payment  =  165.56 
 

Finding the Interest Rate per period and the Nominal Interest Rate  
Present value is known 
To find the interest rate when the present value is known, the formula may be used: 

𝐏𝐫𝐞𝐬𝐞𝐧𝐭 𝐕𝐚𝐥𝐮𝐞
𝑻𝒂𝒃𝒍𝒆 𝟒 𝒇𝒂𝒄𝒕𝒐𝒓  
𝐀𝐧𝐧𝐮𝐢𝐭𝐲 𝐩𝐚𝐲𝐦𝐞𝐧𝐭
Illustration: The present value of an annuity is 2,000 payable at the end of every six months for ten 
years is P30, 000. Find the nominal rate compounded semi‐annually. 

Substitute the value of the following: PV = P30, 000; n = 20, Pmt = P2, 000 

Factor  =  Present Value


 
Annuity payment

Factor  =  30,000
 
20

Factor  =  15 

Follow the line for n = 20 in table 4 to find the value/s of or closest to 15. This is between 2  % and 
3% with factor of 15.227252 and 14.877475 respectively.  

To get the more accurate value of the interest rate, the interpolation method is used: 

  Interest period per period  Table factor    

  2  %  15.227252  (1) 

  x  15.000000  (2) 

Workbook in Math of Investments    26 
 
  3%  14.877475  (3) 

(2)‐(3)  x – 3%  0.122525  (4) 


(1)‐(3)  =  0.349777  (5) 
‐ % 
 

Solve for x from the proportion formed by the differences on lines (4) and (5) 

=  0.122525 
x – 3%  ‐ % 
0.349777 
x – 0.03   =  ‐0.0025 x .350295 
x – 0.03   =  ‐0.000876 
x   =  ‐0.000876 + 0.03 
X  =  0.29124 
The desired value of i is 0.029124 or 2.91% per semi‐annual period. So, the nominal rate per annum 
is 5.82%. 

Finding the term 
Future value is known 

To find the term of an annuity when the future value is known, the formula may be used: 

𝐅𝐮𝐭𝐮𝐫𝐞 𝐕𝐚𝐥𝐮𝐞
𝑻𝒂𝒃𝒍𝒆 𝟑 𝒇𝒂𝒄𝒕𝒐𝒓  
𝐀𝐧𝐧𝐮𝐢𝐭𝐲 𝐏𝐚𝐲𝐦𝐞𝐧𝐭
 

Illustration: If P3, 000 is deposited at the end of each month, how many months will be required for 
the deposits to amount to P122, 000, if the interest is 6% compounded monthly? 

Substitute the value of the following: FV = P122, 000; i = 1/2%; Pmt = P3, 000 

Factor   =  Future Value


 
Annuity Payment
Factor   =  122,000
 
3,000
Factor   =  40.666667 
 

In the ½% column of Table 3, find the two values closest to 40.666667.  This is between n = 37 and 
38 with 40.532785 and 41.735449 factor respectively.  

Workbook in Math of Investments    27 
 
By trial‐and‐error method, we will compute the corresponding future value. The 37 term has future 
value of P121, 598.36 which shows an insufficient payment. Therefore, 38 months is the answer with 
the last deposit of less than P3, 000 to arrive at the desired future value. 

   

Workbook in Math of Investments    28 
 
CHAPTER 4 EXERCISES 
I. ORDINARY ANNUITY 
a. Finding the Size of Payment 

  Future Value  Present Value  Payment  Term  Compound 


Interval  Interest Rate 
1.  1,000    1 year  5 years  10% annually 
2.    7,000  1 month   7 years, 1 month  18% monthly 
3.    6,000  1 quarter  3 years  14% quarterly 
 

b. Finding the Future Value   

  Annuity  Payment  Time  Nominal  Interest  Future Value Of The 


Payment   Frequency  Period  Rate  Compounded  Annuity 
1.  10,000  Every 3 months  4 years  8%  Quarterly       
2.  15, 000  Every year  7 years  16%  Quarterly       
3.  2,000  Every 6 months  2 years  12%  Monthly       
 

c. Finding the Present Value 

  Annuity  Payment  Time  Nominal  Interest  Present Value Of The 


Payment   Frequency  Period  Rate  Compounded  Annuity 
1.  3,000  Every 6 months  7 years  10%  Semiannually       
2.  85,000  Every year  3 years  16%  Quarterly       
3.  10,000  Every month  1 3/4 years  6%  Annually       
 
d. Finding the Interest Rate per Period and the Nominal Rate 

  Future Value  Present Value  Payment  Term  Interest – Compounding Period 


1.  3,160    200 annually  10 years  1 year 
2.    6,642  270 quarterly  7 years  1 quarter 
3.    530  80 annually  8 years  1 year 
 

e. Finding the Term 

  Future Value  Present value  Payment  Compounding Interest Rate 


1.  7,500    600 annually  10% annually 
2.    4,092  120 quarterly  5% quarterly 
3.    18,400  2,000 annually  8 ½ % annually 
 

Workbook in Math of Investments    29 
 
II. WORD PROBLEMS 
1. If Bred borrowed P750, 000 and agreed to repay it by paying P80, 000 at the end of each 
year.  If  the  interest  rate  was  9%  compounded  annually,  how  many  payments  was  he 
required to make? 

2. At  what  nominal  interest  rate  compounded  semiannually  will  an  annuity  of  P2,  200 
payable at the end of every 6 months amount to P25, 300 in 5 years? 
 

3. A debt of P25, 000 was repaid in ten equal quarter payments. If the rate of the interest 
was 7% compounded annually, what was the size of each payment? 

4. Bon Savings and Loans is paying 6% interest compounded annually. How much must be 
deposited now in order to withdraw P4, 000 at the end of each month for 2 years? 
 

5. Bamboo  is  planning  for  his  retirement.  She  deposits  P30,  000  at  the  end  of  each  year 
into  an  account  paying  5%  interest  compounded  annually.  (a)  How  much  would  the 
account be worth after 10 years? (b) How would the account be worth after 20 years? 
(c) When Bamboo retires in 30 years, what will be the total worth of the account? (d) If 
Bamboo found a bank that paid 6% interest compounded annually, rather than 5%, how 
much more would she have in the account after 30 years? 

   

Workbook in Math of Investments    30 
 
Chapter 5 

SIMPLE ANNUITY DUE 
Annuity Due ‐ periodic payments are made at the beginning of each payment interval  
Future Value using the Formula 
 

The formula is the same except that it is multiplied by (1 + i).  
The formula to be used for the future value of annuity due states: 
𝒏
𝟏 𝒊 𝟏
𝐅𝐕 𝑷𝒎𝒕 𝒙 𝒙 𝟏 𝒊  
𝒊
Where: 

FVAD = future value of an annuity due 
Pmt = annuity payment 
          i = interest rate per period 
n = number of periods (years x periods per year) 
 

Using the Table 
Illustration:  Andrey  deposited  P6,  000  at  the  beginning  of  each  month,  for  2  years  at  her  credit 
union.  If  the  interest  rate  was  12%  compounded  monthly,  use  the  table  3  but  with  some 
modifications to calculate the future value of annuity due. 

To  solve,  add  1  period  and  deduct  1.000000  from  the  table  factor  to  get  the  annuity  due  factor. 
Then, substitute the following values: Pmt = P6, 000; Table factor = (28.243200 – 1) 

Future value   =   Annuity payment x table factor 


Future value  =  6,000 x 27.243200 
Future value  =  P163, 459.20 
 
Present Value using the Formula 
The formula is the same except that it is multiplied by (1 + i).  
The formula to be used for the Present value of annuity due states: 
𝒏
𝟏 𝟏 𝒊
𝐅𝐕 𝑷𝒎𝒕 𝒙 𝒙 𝟏 𝒊  
𝒊
Where: 

PVAD = Present value of an annuity due 
Pmt = annuity payment 
          i = interest rate per period 
n = number of periods (years x periods per year) 
 
 
 

Workbook in Math of Investments    31 
 
Using the Table 
Illustration: How much must be deposited now, at 10% compounded semi‐annually, to get yield an 
annuity payment of P20, 000 at the beginning of each period for 7 years? Use the table 4 but with 
some modifications to calculate the Present value of annuity due. 

To  solve,  deduct  1  period  and  add  1.000000  from  the  table  factor  to  get  the  annuity  due  factor. 
Then, substitute the following values: Pmt = P6, 000; Table factor = (9.395973 + 1) 

Present value   =   Annuity payment x table factor 


Present value  =  20,000 x 10.395973 
Present value  =  P207,871.46 
 

 
ADDITIONAL PROBLEMS  
Finding The Annuity Payment when Present value is known 
To find the size of each payment, the formula may be used: 

𝐏𝐫𝐞𝐬𝐞𝐧𝐭 𝐕𝐚𝐥𝐮𝐞
𝑨𝒏𝒏𝒖𝒊𝒕𝒚 𝒑𝒂𝒚𝒎𝒆𝒏𝒕 𝑻𝒂𝒃𝒍𝒆 𝟒 
𝐓𝐚𝐛𝐥𝐞 𝐅𝐚𝐜𝐭𝐨𝐫
Illustration:  A  washing  machine  that  sells  for  P6,  000  can  be  bought  under  the  terms  of  20  equal 
monthly payments starting now. If the money is worth 21% compounded monthly, what is the size 
of each payment?  

Substitute the value of the following: PV = P6, 000; i = 1 ¾ %; n = 19 (20 ‐1) 

Annuity payment  =  Present Value


 
Table Factor
Annuity payment  =  6,000
 
17.046057
Annuity payment  =  351.99 
 

Finding The Annuity Payment when Future value is known 
To find the size of each payment, the formula may be used: 

𝐅𝐮𝐭𝐮𝐫𝐞 𝐕𝐚𝐥𝐮𝐞
𝑨𝒏𝒏𝒖𝒊𝒕𝒚 𝒑𝒂𝒚𝒎𝒆𝒏𝒕 𝑻𝒂𝒃𝒍𝒆 𝟑 
𝐓𝐚𝐛𝐥𝐞 𝐅𝐚𝐜𝐭𝐨𝐫
Illustration: Angelo wishes to receive P20, 000 five years from now. How much must he invest at the 
beginning of each year if the first payment starts now and the interest is 10% compounded annually? 

Workbook in Math of Investments    32 
 
Substitute the value of the following: FV = P20, 000; i = 10%; n = 6 (5 + 1) 

Annuity payment  =  Future Value


 
Table Factor
Annuity payment  =  20,000
 
6.715610
Annuity payment  =  P2,978.14 
 

Finding the Interest Rate per period  
To find the interest rate and the term, the formula may be used: 

(a) When Present value is known 
Step 1   PV = Pmt x Table Factor (Table 4) 

Step 2   PV = Pmt x Table Factor + Pmt (Table 4) 

(b) When Future value is known 
Step 1   FV = Pmt x Table Factor (Table 3) 

Step 2   FV = Pmt x Table Factor + Pmt (Table 3) 

Illustration:  What  is  the  nominal  rate  compounded  quarterly  if  the  present  value  of  an  annuity  of 
P3,000 payable at the beginning of each quarter for 6 years is P58,000? 

Substitute the value of the following: PV = P58, 000; n = 23 (4 x 6 – 1), Pmt = P3, 000 

PV  =  Pmt x Table factor + Pmt 
PV – Pmt  =  Pmt x table factor 
Factor  =  PV Pmt
 
Pmt

Factor  =  58,000 3,000


 
3,000

Factor  (Table 4)  =  18.333333 
 

Follow the line for n = 23 in table 4 to find the value/s of or closest to 18.33. This is between 1  % 
and  2%  with  factor  of  18.544215and  18.292204  respectively.  So,  the  value  of  nominal  rate  is 
between 7 ½ % and 8%. 

To get the more accurate value of the interest rate, the interpolation method is used. 

   

Workbook in Math of Investments    33 
 
Finding the term 
Future value is known 
Illustration:  If  P1,  000  is  deposited  at  the  beginning  of  each  month  at  an  interest  rate  of  9% 
compounded  monthly,  how  many  months  will  be  required  for  the  deposits  to  amount  to  at  least 
P76, 000? 

Substitute the value of the following: FV = P76, 000; i = ¾ %; Pmt = P1, 000 

FV  =  Pmt x Table factor ‐ Pmt 
PV + Pmt  =  Pmt x table factor 
Factor   =  𝐹𝑉 𝑃𝑚𝑡
 
Pmt

Factor   =  76,000 1,000


 
1,000

Factor   =  77 
 

In the ¾ % column of Table 3, find the two values closest to 77. This is between n = 61 and 62with 
76.989818 and 78.567242 factor respectively.  

By  trial‐and‐error  method,  we  will  compute  the  corresponding  future  value  of  annuity  due. 
Therefore, 62, the larger number, is employed. Actual period is 5 years and 1 month (n = 62 – 1).  

   

Workbook in Math of Investments    34 
 
CHAPTER 5 EXERCISES 
 
I. ANNUITY DUE 
a. Finding the Future Value   

  Annuity  Payment  Time  Nominal  Interest  Future value of the 


payment   frequency  period  Rate  compounded  Annuity 
1.  4,000  Every 6 months  12 years  10%  Semiannually       
2.  44,000  Every 6 months  8 years  16%  Monthly       
3.  2,000  Every year  25 years   5%  Annually       
 

b. Finding the Present Value 

  Annuity  Payment  Time  Nominal  Interest  Present Value Of The 


Payment   Frequency  Period  Rate  Compounded  Annuity 
1.  14,000  Every year  10 years  11%  Quarterly       
2.  40,000  Every month  7 years  18%  Monthly       
3.  70,000  Every 6 months  12 years  8%  Semiannually       
 
c. Finding the Unknown Values 

  Future Value  Present Value  Payment  Term  Compound Interest Rate 

1.  5,800    P200 monthly  2 years  ? monthly 


2.     ?      450  30 quarterly  5 years  ? quarterly 
3.    3,800  250 semiannually     ?      6% semiannually 
4.  6,000    ? semiannually  7 ½ years  4% semiannually 
5.  3,200    120 monthly     ?      7% monthly 
 

 
II. WORD PROBLEMS 
1. Cora  is  paying  6%  interest  compounded  quarterly.  Find  the  future  value  of  P1,  000, 
deposited at the beginning of every 3 months, for 5 years. 

2.  If P10, 000 must be withdrawn at the beginning of each 3‐month period for the next 3 
years,  how  much  must  be  deposited  now,  at  6%  compounded  quarterly,  to  yield  the 
annuity payment needed? 
 

3. A  house  that  sells  for  P900,  000  can  be  purchased  under  the  terms  requiring  100 
monthly payments. Assume  that the  first payment begins now  and the interest is 12% 
compounded monthly. What is the size of each payment? 

Workbook in Math of Investments    35 
 
4. Chin made a New Year’s Resolution to put P1, 000 into the bank at the beginning of each 
month, beginning January 2014. If the bank pays 6% interest compounded monthly on 
the last day of each month, how much will Chin have one year later? 
 

5. The monthly rent for the one‐bedroom apartment in JV Hotel is P15, 000, payable at the 
beginning of the month. If the current interest rate is 9%, what would be a fair amount 
to charge someone if they wish to pay their yearly rental in advance? 

             
 
   

Workbook in Math of Investments    36 
 
Chapter 6 

SIMPLE DEFERRED ANNUITY AND SIMPLE PERPETUITY 
   
 
DEFERRED ANNUITY 
  A deferred annuity is an annuity in which the first periodic payment is made several periods 
after the beginning of the annuity.  
 
Period of deferment ‐ The period between now and the beginning of the term of the annuity. 
 
Future Value Using the Table 
  The  future  value  of  the  deferred  annuity  is  the  same  as  the  future  value  of  the  ordinary 
annuity. Thus, the future value of an ordinary annuity table is used. 
         
        FV Pmt x Table factor 
 
Illustration: What is the future value of an annuity of P3,000 payable at the end of each quarter for 
six  payments  with  the  first  payment  due  at  the  end  of  nine  months.  The  interest  rate  is  5% 
compounded quarterly. 
 
Solution: 
FV Pmt x Table factor
FV P3,000 x 6.190654
FV P18,571.96

Present Value Using the Table 
  In finding the present value of an annuity, a series of steps is followed. Let d = the number of 
deferred payment intervals.   
        PV Pmt x Table factor 
 
Illustration: What amount should you invest now if you want to receive payments of P3,000 at the 
end  of  each  quarter  for  six  payments  with  the  receipt  of  the  first  payment  at  the  end  of  nine 
months? The interest rate is 5% compounded quarterly. 
 
Solution: 
Given, Pmt P3,000; i 1 ¼%; n 6; d 2 
 
Step  1.  Find  the  present  value  assuming  that  payments  were  made  even  during  the  period  of 
deferment.    
Using d + n as the total number of compounding periods, the period to be used in locating the table 
factor is 8 (2+6). 
PV Pmt x Table factor
PV P3,000 x 7.568124
PV P22,704.37

Workbook in Math of Investments    37 
 
Step 2. Find a present value assuming only d or 2 – the period of deferment as the total number of 
compounding periods.  
PV Pmt x Table factor
PV P3,000 x 1.963115
PV P5,889.35

Step 3. Subtract the present value in step 2 from the present value in step 1. The difference is the 
present value of the deferred annuity.  
        PV P22,704.37 – P5,889.35
PV P16,815.02

Alternatively, the final present value may be solved as: 
        PV Pmt x Difference in table factors
PV P3,000 7.568124-1.963115
PV P16,815.02
 
ADDITIONAL PROBLEMS  
 
Finding the Annuity Payment when Present Value is Known 
 
Illustration:  Belle purchased on account a computer for P90,000 on June 1, 2015 and agreed to pay 
in 14 annual payments plus interest at 9.5% compounded annually, with the first payment due on 
June 1, 2018. What is the size of each payment? 
 
Solution: 
Given, PV P90,000; i 9.5%; n 14; d 2
𝑃𝑉
𝑃𝑚𝑡
Table factor
90,000
𝑃𝑚𝑡
8.062260 1.747253
𝑃𝑚𝑡 P14,251.77
 
Finding the Term when Present Value is Known 
 
Illustration:    If  P10,000  is  deposited  at  the  end  of  each  month  and  the  interest  rate  is  6% 
compounded monthly, how many months will be required for the deposits to equal a present value 
of P500,000? The first deposit is made at the end of six months. 
 
 
 
Solution: 
Given,   PV P500,000; Pmt P10,000; i .5%; d 5 
Find:      Factor AD; Factor d n
Factor AD PV/Pmt
Factor AD 500,000/10,000
Factor AD 50
Factor d n Factor AD Factor d

Workbook in Math of Investments    38 
 
Factor d n 90 4.925866
Factor d n 94.925866

To find n; find the first entry greater than 94.925866 in the .5% column of Table 4. That is the factor 
for 130 periods which is 95.421606. Therefore, n= 130‐5 = 125. 
 
PERPETUITY 
  A perpetuity is an annuity where the periodic payments continue indefinitely. 
 
Simple  ordinary  perpetuity ‐ a perpetuity in which  the  periodic payments are  made at  the  end of 
each interest period. Present value of which is computed as  
 
    PV∞ I/R

  Where:
I = periodic payment made at the end of each period and 
                 R = interest rate per period  
 
Illustration:  If  money  is  worth  8%  compounded  quarterly,  compare  the  present  values  of  the 
following: 
    1. an annuity of P2,000 payable quarterly for 50 years; 
    2. an annuity of P2,000 payable quarterly for 100 years; 
    3. a perpetuity of P2,000 payable quarterly. 
Solution: 
1. Pmt 2,000; i 2%; n 200
PV Pmt x Table factor
PV P2,000 x 49.047345
PV P98,094.69

2. Pmt 2,000; i 2%; n 400


PV Pmt x Table factor
PV P2,000 x 49.981849
PV P99,963.70

3. I 2,000; r 2%;
PV∞ I/R
PV∞
2,000/2%
PV∞
P100,000
Observe that the present value of an annuity, whose term is increasing, gets closer to the
present value of perpetuity.
 
 
 
 

Workbook in Math of Investments    39 
 
Simple perpetuity due ‐ is a perpetuity in which the periodic payments are made at the beginning of 
each interest period. Present value of which is computed as  
 
    PV∞ I/R I

  Where:
I = periodic payment made at the beginning of each period and 
                 R = interest rate per period  
 
Illustration:  at  2%  interest  per  quarter,  what  will  be  the  present  value  of  the  simple  perpetuity  if 
payment of P2,000 is made at the beginning of each quarter? 
 
Solution: 
Given,  I 2,000; r 2%;
PV∞ I/R
PV∞ 2,000/2% 2, 000
PV∞ P102,000
 
 

   

Workbook in Math of Investments    40 
 
CHAPTER 6 EXERCISES 
I. Deferred Annuity 
 
Finding the unknown values. 
     Future        Present  Payment  Number of  Period of  Compound 
     Value  Value  Payments  Deferment  Interest Rate 

1.  P1,600  125 quarterly  ?  7  14% quarterly 

2.  P1,000  ? annually  10  3  10% annually 

3.       ?  ?  600 monthly  14  12  18% monthly 

 
 
 
 
Word Problems. 
 
1. Find the future value and the present value of an annuity of P20,000 payable at the  
end  of  every  three  months  for  20  payments.  Interest  rate  is  11%  compounded 
quarterly. The   first payment is due at the end of six years. 
 
 
 
2. A set of computer costs P70,000 if paid in cash. On the installment plan, a purchaser 
should  pay  P20,000  down  payment  and  the  balance  to  be  paid  in  10  quarterly 
installments,  the  first  due  at  the  end  of  the  first  year  after  purchase.  If  money  is 
worth 15% compounded quarterly, determine the size of each payment. 
 
 
3. Baker  Company  owns  a  plot  of  land  on  which  buried  toxic  wastes  have  been 
discovered.  Since  it  will  require  several  years  and  a  considerable  sum  of  money 
before  the  property  is  fully  detoxified  and  capable  of  generating  revenues,  Baker 
wishes  to  sell  the  land  now.  It  has  located  two  potential  buyers:    Buyer  A,  who  is 
willing  to  pay  $320,000  for  the  land  now,  and  Buyer  B,  who  is  willing  to  make  20 
annual payments of $50,000 each, with the first payment to be made 5 years from 
today. Assuming that the appropriate rate of interest is 9%, to whom should Baker 
sell the land? 
 
 
4. Bethany borrowed P65,000 at 7% interest compounded quarterly, and agreed to pay  
the loan in quarterly payments of P5,000 each. The first payment is due in two years. 
Find the number of payments. 
 
 

Workbook in Math of Investments    41 
 
II. Perpetuity 
Word Problems. 
 
1. It costs P40,000 at the end of each year to maintain a section of Diversion road in  
Davao   City. If money is worth 10% compounded annually, find the present value of 
a simple perpetuity. 
 
 
 
2. What will be the present value of the simple perpetuity if payment of P4,000 is  
made at the beginning of each quarter at 5% interest per quarter. 
 
 
 
3. Find the present value of a simple perpetuity of P15,000 payable semi‐annually if the 
interest rate is 2% per six months and the first payment is due (a) six months from 
now, and (b) now.     
 
 
 
 
 
 
 
 
 
 
 
 
 
 
 
 
   

Workbook in Math of Investments    42 
 
            Chapter 7 

SINKING FUND AND AMORTIZATION 
   
 
SINKING FUND 
  Putting up a sinking fund means setting aside equal amounts of money at the end of each 
period, at a compound interest, for the purpose of saving for a future project or obligation. 
   
Sinking Fund Payment Using the Table     
𝐅𝐮𝐭𝐮𝐫𝐞 𝐯𝐚𝐥𝐮𝐞 𝐨𝐟 𝐭𝐡𝐞 𝐬𝐢𝐧𝐤𝐢𝐧𝐠 𝐟𝐮𝐧𝐝
𝐒𝐢𝐧𝐤𝐢𝐧𝐠 𝐟𝐮𝐧𝐝 𝐩𝐚𝐲𝐦𝐞𝐧𝐭  
𝐅𝐮𝐭𝐮𝐫𝐞 𝐯𝐚𝐥𝐮𝐞 𝐭𝐚𝐛𝐥𝐞 𝐟𝐚𝐜𝐭𝐨𝐫
             
Illustration: What sinking fund payment is required at the end of each 6‐month period at 6% interest 
compounded semiannually, to amount to P120,000 in 4 years? 
 
Solution: 
Future value of the sinking fund
Sinking fund payment  
Future value table factor
Sinking fund payment P120,000 / 8.892336
Sinking fund payment P13,494.77

Sinking Fund Payment Using the Formula 
𝒊
𝐒𝐢𝐧𝐤𝐢𝐧𝐠 𝐟𝐮𝐧𝐝 𝐩𝐚𝐲𝐦𝐞𝐧𝐭 𝑭𝑽 𝐱 𝒏
 
𝟏 𝒊 𝟏
where
FV amount needed in the future
i interest rate per period
n number of periods

Illustration: HBS Corporation needs P1,200,000 in 6 years to pay off a bond issue. What sinking fund 
payment is required at the end of each month, at 12% interest compounded monthly, to meet this 
financial obligation? 
 
Solution: 
    Sinking fund payment 𝐹𝑉 x
 
.01
Sinking fund payment 1,200,000 x  
1 .01 1
     
.
                     Sinking fund payment 1,200,000 x
.
Sinking fund payment P11,460.23 
 
 
 

Workbook in Math of Investments    43 
 
Sinking Fund Schedule 
 
Illustration 1: A P40,000 debt is to be repaid at the end of 1.5 years. Interest charged is 15% payable 
at  the  end  of  every  3  months.  The  debtor  established  a  sinking  fund  that  earns  12%  interest 
compounded quarterly. Construct a sinking fund schedule. 
 
Solution:  
 
  Sinking fund payment 𝐹𝑉 x  
 
.03
Sinking fund payment 40,000 x  
1 .03 1
 
    Sinking fund payment 40,000 x .1545975 

Sinking fund payment P6,183.90  


 
(1) (2) (3) (4) (5) (6)
At end of period Interest Income Periodic Deposit Periodic Sinking Fund Book Value
on Sinking Fund in Fund Increase in Fund Accumulated P40,000 - (5)
3% x (5)* (2)+(3) (4)+(5)*

1 6,183.90 6,183.90 6,183.90 33,816.10


2 185.52 6,183.90 6,369.42 12,553.32 27,446.68
3 376.60 6,183.90 6,560.50 19,113.82 20,886.18
4 573.41 6,183.90 6,757.31 25,871.13 14,128.87
5 776.13 6,183.90 6,960.03 32,831.16 7,168.84
6 984.93 6,183.90 7,168.83 40,000.00 0.00
Total 2,896.60 37,103.40 40,000.00

 
Alternatively, the information in the columns of the sinking fund can be obtained without 
constructing a schedule. 
 
Illustration 2: Given the same information in Illustration 1, find (a) the amount in the sinking fund at 
the end of the fourth period, (b) the sinking fund interest income for the fifth payment period, and 
(c) the book value of the debt at the end of the fourth period. 
 
(a)  The amount in the sinking fund at the end of the fourth period is the future value an annuity of     
       P6,183.90 payable quarterly at 12% compounded quarterly for four periods. 
 
        FV Pmt x Table factor
FV P6,183.90 x 4.183627
FV P25,871.13

(b)  The principal at the beginning of the fifth payment period is the amount in the sinking fund at 
the      
       end of the fourth period. 
        Interest income P25,871.13 x .03

Workbook in Math of Investments    44 
 
Interest income P776.13

(c)  The book value is the net obligation, which equals the original debt less the accumulated amount 
in   
       the fund at that time. 
 
        Book value 40,000 – 25,871.13
Book value P14,128.87

Illustration 3: Using illustration 1, except that interest rates for different periods vary. During the 
first and second periods, interest rate was 3%, 3 ½ % during the third and fourth periods , and 2 ½% 
during the fifth and sixth periods.  

(1) (2) (3) (4) (5) (6) (7)


At end of Interest Actual Interest Adjusted Periodic Sinking Fund
period Income on Interest Discrepancy Deposit Increase in Accumulated
Sinking Income Schedule Fund (3) +
Fund 3% x P6,183.90 + (5)
(5)* (4)
1 - 6,183.90 6,183.90
2 185.52 185.52 - 6,183.90 6,369.42 12,553.32
3 376.60 439.37 (62.77) 6,121.13 6,560.50 19,113.82
4 573.41 668.98 (95.57) 6,088.33 6,757.31 25,871.13
5 776.13 646.78 129.36 6,313.26 6,960.03 32,831.16
6 984.93 820.78 164.16 6,348.06 7,168.83 40,000.00
Total 2,896.60 2,761.42 135.18 37,238.58 40,000.00

AMORTIZATION 
  Amortization is a method of repaying debt, the principal and interest included, usually by a 
series of equal payments at equal interval of time. 
 
Amortization Payment Using the Table     
𝐎𝐫𝐢𝐠𝐢𝐧𝐚𝐥 𝐚𝐦𝐨𝐮𝐧𝐭 𝐨𝐟 𝐨𝐛𝐥𝐢𝐠𝐚𝐭𝐢𝐨𝐧
𝐀𝐦𝐨𝐫𝐭𝐢𝐳𝐚𝐭𝐢𝐨𝐧 𝐩𝐚𝐲𝐦𝐞𝐧𝐭  
𝐏𝐫𝐞𝐬𝐞𝐧𝐭 𝐯𝐚𝐥𝐮𝐞 𝐭𝐚𝐛𝐥𝐞 𝐟𝐚𝐜𝐭𝐨𝐫
Illustration:  What  amortization  payments  are  required  each  month,  at  15%  interest,  to  pay  off  a 
P15,000 loan in 2 years? 

Original amount of obligation


Amortization payment  
Present value table factor

15,000
Amortization payment  
20.624235

Amortization payment P 727.30 


 
 

Workbook in Math of Investments    45 
 
Amortization Payment Using the Formula 
 
Illustration: Tyndale purchased on account a machine worth P50,000. What amortization payment is 
required each month, at 18% interest to pay off his debt in 3 years? 
 
𝑖
Amortization payment Present value x  
1 1 𝑖
.
                               Amortization payment 50,000 x  
.
.
                               Amortization payment 50,000 x  
.

                               Amortization payment P1,807.62 


 
All Periodic Payments are Equal 
 
Illustration: A P40,000 debt is to be amortized by equal payments at the end of every quarter for 1.5 
years.  Interest  charged  is  12%  compounded  quarterly.  The  debtor  established  a  sinking  fund  that 
earns 12% interest compounded quarterly. Construct an amortization schedule. 
𝑖
Amortization payment Present value x  
1 1 𝑖
.
                               Amortization payment 40,000 x  
.
.
                               Amortization payment 40,000 x  
.

                               Amortization payment P7,383.90 


(1) (2) (3) (4) (5)
Period Outstanding Interest Due at Equal Payment Portion of
Principal at End of Period at End of Each Principal
Beginning of (2) x 3% Period Reduced by
Each Period Each Payment
(2)-(5)** (4)-(3)
1 40,000.00 1,200.00 7,383.90 6,183.90
2 33,816.10 1,014.48 7,383.90 6,369.42
3 27,446.68 823.40 7,383.90 6,560.50
4 20,886.18 626.59 7,383.90 6,757.31
5 14,128.87 423.87 7,383.90 6,960.03
6 7,168.84 215.07 7,383.90 7,168.83
Total 4,303.40 44,303.40 40,000.00
 
  Alternatively, the outstanding principal can be obtained using the present value of an 
annuity. For example, the outstanding principal after the fourth payment is made is the present 
value of an annuity formed by the two remaining unpaid payments. 
 
    Outstanding principal Periodic payment x Present value table factor
    Outstanding principal 7,383.90 x 1.913470
Outstanding principal P14,128.87

Workbook in Math of Investments    46 
 
     
All Periodic Payments Except the Final Payment are Equal 
 
Illustration: A P60,000 debt is to be discharged by payments of P10,000 at the end of every month. 
Interest charged is 12% compounded monthly. Construct an amortization schedule.  
 
(1) (2) (3) (4) (5)
Period Outstanding Interest Due at Equal Payment Portion of
Principal at End of Period at End of Each Principal
Beginning of (2) x 1% Period Reduced by
Each Period Each Payment
(2)-(5)** (4)-(3)

1 60,000.00 600.00 10,000.00 9,400.00


2 50,600.00 506.00 10,000.00 9,494.00
3 41,106.00 411.06 10,000.00 9,588.94
4 31,517.06 315.17 10,000.00 9,684.83
5 21,832.23 218.32 10,000.00 9,781.68
6 12,050.55 120.51 10,000.00 9,879.49
7 2,171.06 21.71 2,192.77 2,171.06
Total 2,192.77 62,192.77 60,000.00  
 
 

   

Workbook in Math of Investments    47 
 
CHAPTER 7 EXERCISES 
I. Finding the Unknown. 
 
a. Finding the sinking fund payment. 
Sinking Fund  Payment  Time   Nominal   Interest  Future  
Payment      Frequency  Period  Rate  Compounded  Value 

1.__________   every 6 months 8 years  10%  semi‐annually  500,000 

2.__________  every year  14 years  9%  annually  2,500,000 

3.__________  every 3 months  5 years  12%  quarterly  15,000 

 
 
In eaI 
b. Finding the unknown. 
In each of the following problems, find (a) the interest payment for each interest period, (b) 
the size of deposits to the sinking fund, (c) the amount in the sinking fund at the end of the 
nth  period,  (d)  the  book  value  of  the  debt  at  the  end  of  the  nth  period,  and  (e)  the  sinking 
fund schedule interest income for the (n + 1)th payment period. Do not construct a sinking 
fund schedule in finding your answers. 

Debt      Interest Rate  No. of Deposits  Interest Rate on  nth Period 


on the Debt  in Sinking Fund  Sinking Fund 
1.   P6,000  24%, monthly  20, monthly  18%, monthly  6th   

2.   P10,000  6%, semi‐annuall 8, semi‐ 5% semi‐ 5th   


annually  annually 
3.   P450  7%, annually   5, annually  10%, annually  3rd   
 

c. Finding the amortization payment. 
 
  Loan  Payment  Term of Loan  Nominal   Present Value 
Payment      Period  Rate  (Amount of 
  Loan) 
 
1.__________  every month  1 ¾ years  18%  100,000 
 
  2.__________  every year  12 years  9%  300,000 

  3.__________  every month  1 ½ years  12%  8,500 


 
 
 
 
 

Workbook in Math of Investments    48 
 
d. Finding the unknown. 
In  each  of  the  following  problems,  find  (a)  the  number  of  payments,  (b)  the  outstanding 
principal  at  the  indicated  time,  (c)  the  interest  and  the  principal  included  in  the  next 
payment after the indicated time in (b), and (d) the size of the final payment and the total 
cash payments. Do not construct an amortization schedule in finding your answers. 

Debt      Payments  Compound Interest  Required Outstanding 


Rate  Principal 

1.   P4,000  P200 every 3 months  8%, quarterly  After 20th payment 

2.   P5,000  P1,000 every month  24%, monthly  After 3rd payment 

3.   P1,400  P50 every month  12%, monthly  After 6th payment 

 
II. Word Problems. 
1. Ana wants to accumulate P80,000 in 5 years for a trip. If her bank is paying 12%  
interest compounded quarterly, how much must Ana deposit at the end of each 3‐
month period to reach her desired goal? 
2. A  condominium  association  wants  to  establish  a  sinking  fund  to  accumulate 
2,500,000  in  3  years  to  repair  the  concrete  roof.  The  fund  earns  9%  interest 
compounded monthly.  If there are 200 units in the condominium, how much should 
each unit owner be assessed each month as a fair contribution to the fund? Assume 
that all units are of equal size and with equal assessment. 
 
3. A  P50,000  debt  is  to  be  repaid  at  the  end  of  one  year.  The  debtor  establishes  a 
sinking  fund that earns 8% interest compounded quarterly. Construct a sinking fund 
schedule. 
 
4. Referring to no. 1 above, assume that the interest rate on the sinking fund was 8% 
compounded  quarterly  during  the  first  ½  years  and  12%  compounded  quarterly 
during   the second ½ year. Construct a sinking fund schedule. 
 
5. Find the monthly payment of an auto loan of P200,000 to be amortized over a 5‐
year period at a rate of 9%. 
 
6. Khan purchased a new motorcycle for P130,000. He made a P20,000 down payment
and financed the balance at his bank for 7 years. What amortization payments are
required every 3 months, at 16% interest, to pay off the boat loan? 

7. A  debt  of  P80,000 is to amortized with P25,000 being paid at the end of every six
months. The interest rate is 6% compounded semi-annually. Construct an
amortization schedule.

8. A debt of P40,000 is to be amortized with P8,000 being paid at the end of each
quarter. The interest rate is 16% compounded quarterly. Construct an amortization
schedule.

     
   

Workbook in Math of Investments    49 
 
Chapter 8 
DEPRECIATION 
   
   
Definition of terms 
 
Original cost or total cost – this amount includes items such as purchase price, freight, handling and 
set‐up  charges  and  other  costs  that  are  directly  attributable  in  bringing  the  asset  to  its  present 
location and condition. 
 
Salvage value/ Scrap value/ Residual value/ Trade‐in value– this amount is the estimated value of 
the asset at the time it is taken out if service. 
 
Estimated useful life – this is the length of time that an asset is expected to generate revenue. 
 
Depreciation ‐ is the systematic allocation of the cost of the assets over their useful life. This is done 
to properly match expense with revenue earned. 
 
 
METHODS USED FOR FINANCIAL STATEMENT REPORTING 
     
Straight‐Line Method 
  It  is  simple  and  is  more  widely  used  than  any  other  method.  It  provides  a  uniform  annual 
charge and is calculated as: 
 
𝐓𝐨𝐭𝐚𝐥 𝐝𝐞𝐩𝐫𝐞𝐜𝐢𝐚𝐛𝐥𝐞 𝐜𝐨𝐬𝐭
𝐃𝐞𝐩𝐫𝐞𝐜𝐢𝐚𝐭𝐢𝐨𝐧  
𝐄𝐬𝐭𝐢𝐦𝐚𝐭𝐞𝐝 𝐮𝐬𝐞𝐟𝐮𝐥 𝐥𝐢𝐟𝐞
 
Illustration  1:  A  contractor  purchased  a  gantry  crane  for  P250,000.  Freight  and  insurance  charges 
amounted to P18,000; customs’ broker’s fees, P8,500; taxes, permits and other expenses, P25,000. 
The contractor estimates the life of the gantry crane to be 6 years with a salvage value of P20,000. 
(a) Determine the annual depreciation charge and (b) prepare a depreciation schedule. 
 
Total depreciable cost
Depreciation  
Estimated useful life
 
, , , , ,
        Depreciation  
 
,
        Depreciation  
 
Depreciation P56,300 
 

Workbook in Math of Investments    50 
 
Depreciation Schedule

Annual  Accumulated 
End of Year Depreciation Depreciation Book Value
          301,500 
1               56,300               56,300         245,200
2               56,300             112,600         188,900
3               56,300             168,900         132,600
4               56,300             225,200            76,300
5               56,300             281,500            20,000

 
Sum‐of‐the‐Years’ Digits Method 
  The  basic  assumption  for  this  method  is  that  the  value  of  the  property  decreasing  at  a 
decreasing  rate.  It  provides  very  rapid  depreciation  during  the  early  years  of  its  useful  life.  To 
determine the depreciation, the following equations will be used: 
 
𝒏 𝒏 𝟏
𝑺𝒀𝑫  
𝟐
 
𝐘𝐞𝐚𝐫𝐬 𝐨𝐟 𝐮𝐬𝐞𝐟𝐮𝐥 𝐥𝐢𝐟𝐞 𝐫𝐞𝐦𝐚𝐢𝐧𝐢𝐧𝐠
𝑺𝒀𝑫 𝐝𝐞𝐩𝐫𝐞𝐜𝐢𝐚𝐭𝐢𝐨𝐧 𝐫𝐚𝐭𝐞 𝐟𝐫𝐚𝐜𝐭𝐢𝐨𝐧  
𝐒𝐮𝐦 𝐨𝐟 𝐭𝐡𝐞 𝐝𝐢𝐠𝐢𝐭𝐬 𝐨𝐟 𝐭𝐡𝐞 𝐮𝐬𝐞𝐟𝐮𝐥 𝐥𝐢𝐟𝐞
 
Illustration 2: Refer to Illustration 1. Determine the depreciation charges per year using SYD. Prepare 
a depreciation schedule. 

 
5 5 1
𝑆𝑌𝐷 15 
2
     
 
Years of useful life remaining
𝑆𝑌𝐷 depreciation rate fraction  
Sum of the digits of the useful life
 
Year SYD Depreciation rate fraction
1 5/15
2 4/15
3 3/15
4 2/15
5 1/15

Workbook in Math of Investments    51 
 
Depreciation Schedule
End of  Total  Depreciation  Annual  Accumulated  Book Value
Year Depreciation Rate Fraction Depreciation Depreciation
          301,500
1           281,500 5/15             93,833             93,833           207,667
2           281,500 4/15             75,067           168,900           132,600
3           281,500 3/15             56,300           225,200             76,300
4           281,500 2/15             37,533           262,733             38,767
5           281,500 1/15             18,767           281,500             20,000

Declining‐Balance Method 
  The depreciation cost in any year is a constant ratio of the book value at the beginning of the 
year. Salvage value, if any, is ignored. The most frequently used multiples are 1.25, 1.5 and 2, known 
as 125% declining balance, 150% declining balance and 200% double‐declining balance.  
To calculate for the declining balance rate, the following equation is used: 

𝟏
𝐃𝐞𝐜𝐥𝐢𝐧𝐢𝐧𝐠 𝐛𝐚𝐥𝐚𝐧𝐜𝐞 𝐫𝐚𝐭𝐞 𝐱 𝐌𝐮𝐥𝐭𝐢𝐩𝐥𝐞 
𝐔𝐬𝐞𝐟𝐮𝐥 𝐥𝐢𝐟𝐞
 
Illustration 3: JCF Shipping bought an equipment worth P200,000. It is estimated to have a useful life 
of  5  years  and  a  salvage  value  of  P20,000.  Determine  the  depreciation  charges  and  prepare  a 
depreciation schedule using the double‐declining balance method. 
 
1
Declining balance rate x 2 
5
 
Declining balance rate 40% 
 
Depreciation Schedule
End of  Beginning  Depreciation  Annual  Accumulated  Ending Book 
Year Book Value Rate Depreciation Depreciation Value
          200,000
1           200,000 40%             80,000             80,000           120,000
2           120,000 40%             48,000           128,000             72,000
3             72,000 40%             28,800           156,800             43,200
4             43,200 40%             17,280           174,080             25,920
5             25,920 40% *5,920           180,000             20,000

 
*In year 5, although the estimated depreciation is P10,368 (25,920 x 40%), the allowable 
depreciation is    
  limited to P5,920, because the book value has already reached the P20,000 salvage value. At this 
point,   
  depreciation is complete. 
 
 
 
 

Workbook in Math of Investments    52 
 
Units‐of‐Production Method 
 
  This method accounts for depreciation on the basis of actual service rendered or actual units 
produced. To calculate depreciation using this method, the depreciation per unit is first determined.  
 
Cost salvage value
Depreciation per unit  
units of useful life
 
 
Illustration  4:  A  textile  company  purchased  an  equipment  for  P100,000  with  a  salvage  value  of 
P20,000. For depreciation purposes, the equipment is expected to have a useful life of 5,000 hours. 
From  the  following  estimate  of  hours  of  use,  prepare  a  depreciation  schedule  for  the  equipment 
using the units‐of‐production method. 
 
          Year    Hours of Use 
                1         1,500 
                 2         1,200 
                3         2,000 
                4            500 
 
Cost salvage value
Depreciation per unit  
units of useful life
 
100,000 20,000
Depreciation per unit  
5,000
         
Depreciation per unit P16 
 
Depreciation Schedule
End of  Depreciation  Hours Used Annual  Accumulated  Book Value
Year per hour Depreciation Depreciation
          100,000
1                     16               1,500             24,000             24,000             76,000
2                     16               1,200             19,200             43,200             56,800
3                     16               2,000             32,000             75,200             24,800
4                     16                   500 *4,800             80,000             20,000

 
              *Maximum allowable to reach the salvage value. 
   

Workbook in Math of Investments    53 
 
COMPOUND INTEREST METHODS 
 
Annuity Method 
  This  method  accounts  for  the  periodic  depreciation  charges  as  equal  and  includes  the 
interest  on  the  book  value  for  each  operating  period.  The  periodic  book  value  is  assumed  to  be 
earning  the  same  interest  as  the  amount  would  earn  if  it  were  invested  elsewhere.  The  following 
equations will be used to determine annual depreciation charges. 
 
    𝟏. 𝐏𝐫𝐞𝐬𝐞𝐧𝐭 𝐯𝐚𝐥𝐮𝐞 𝐨𝐟 𝐬𝐚𝐥𝐯𝐚𝐠𝐞 𝐯𝐚𝐥𝐮𝐞 𝐒𝐚𝐥𝐯𝐚𝐠𝐞 𝐯𝐚𝐥𝐮𝐞 𝐱 𝐭𝐚𝐛𝐥𝐞 𝐟𝐚𝐜𝐭𝐨𝐫 
 
𝟐. 𝐏𝐫𝐞𝐬𝐞𝐧𝐭 𝐯𝐚𝐥𝐮𝐞 𝐨𝐟 𝐝𝐞𝐩𝐫𝐞𝐜𝐢𝐚𝐛𝐥𝐞 𝐜𝐨𝐬𝐭
𝐂𝐨𝐬𝐭 𝐩𝐫𝐞𝐬𝐞𝐧𝐭 𝐯𝐚𝐥𝐮𝐞 𝐨𝐟 𝐬𝐚𝐥𝐯𝐚𝐠𝐞 𝐯𝐚𝐥𝐮𝐞 
 
𝐏𝐫𝐞𝐬𝐞𝐧𝐭 𝐯𝐚𝐥𝐮𝐞 𝐨𝐟 𝐝𝐞𝐩𝐫𝐞𝐜𝐢𝐚𝐛𝐥𝐞 𝐜𝐨𝐬𝐭
    𝟑. 𝐀𝐧𝐧𝐮𝐚𝐥 𝐝𝐞𝐩𝐫𝐞𝐜𝐢𝐚𝐭𝐢𝐨𝐧 =   
𝐓𝐚𝐛𝐥𝐞 𝐟𝐚𝐜𝐭𝐨𝐫
 
Illustration 5:  JCF Company bought a machine worth P11,000. It is estimated to have a useful life of 
5  years  and  a  salvage  value  of  P1,200.  Determine  the  depreciation  charges  and  prepare  a 
depreciation  schedule  using  the  double‐declining  balance  method.  Assume  that  the  effective 
interest rate is 6%. 
 
𝟏. Present value of salvage value Salvage value x table factor 
 
Present value of salvage value 20,000 x .747258 P14,945.16 
 
2. Present value of depreciable cost Cost present value of salvage value 
 
Present value of depreciable cost 100,000 14,945.16 P85,054.84

3. Annual depreciation =   
 
, .
Annual depreciation =   = P20,191.71 
.
 
 
Depreciation Schedule
(1) (2) (3) (4) (5) (6)
End of Year Annual  Interest Income  Net  Accumulated  Book Value
Depreciation (6) x 6% Depreciation  Depreciation
Charges
               100,000
1             20,191.71               6,000.00             14,191.71             14,191.71             85,808.29
2             20,191.71               5,148.50             15,043.21             29,234.92             70,765.08
3             20,191.71               4,245.90             15,945.81             45,180.73             54,819.27
4             20,191.71               3,289.16             16,902.55             62,083.28             37,916.72
5             20,191.72               2,275.00             17,916.72             80,000.00             20,000.00
 
Workbook in Math of Investments    54 
 
Sinking Fund Method 
  This method assumes that a sinking fund is established for the purpose of replacing an asset 
at the end of its useful life. It accounts for the periodic depreciation charges as exactly the same as 
the periodic increases (including the periodic deposit and interest) in the sinking fund. The following 
equations will be used to determine annual depreciation charges and periodic deposit in fund. 
 
𝟏. 𝐓𝐨𝐭𝐚𝐥 𝐝𝐞𝐩𝐫𝐞𝐜𝐢𝐚𝐭𝐢𝐨𝐧 𝐚𝐭 𝐭𝐡𝐞 𝐞𝐧𝐝 𝐨𝐟 𝐮𝐬𝐞𝐟𝐮𝐥 𝐥𝐢𝐟𝐞
𝐂𝐨𝐬𝐭 𝐒𝐚𝐥𝐯𝐚𝐠𝐞 𝐯𝐚𝐥𝐮𝐞  
 
𝐓𝐨𝐭𝐚𝐥 𝐝𝐞𝐩𝐫𝐞𝐜𝐢𝐚𝐭𝐢𝐨𝐧
    𝟐. 𝐀𝐧𝐧𝐮𝐚𝐥 𝐝𝐞𝐩𝐨𝐬𝐢𝐭 =   
𝐓𝐚𝐛𝐥𝐞 𝐟𝐚𝐜𝐭𝐨𝐫
 
Illustration 6:  Assume that the effective interest rate is 6%. Use the data in Illustration 5 to find the 
annual depreciation charges by the sinking fund method. Prepare a depreciation schedule. 
 
 
1. Total depreciation at the end of useful life Cost Salvage value  

Total depreciation at the end of useful life 100,000 20,000 P80,000 


 
2. Annual deposit =   
 
,
  Annual deposit  P14,191.71 
.
 
Depreciation Schedule
(1) (2) (3) (4) (5) (6)
End of Year Peridic Deposit  Interest Income  Peridic Increase  Accumulated  Book Value
in Fund from Sinking  in Fund =  Sinking Fund = 
Fund (5) x 6% Annual  Accumulated 
Depreciation  Depreciation
Charges (2) + (3)

               100,000
1             14,191.71             14,191.71             14,191.71             85,808.29
2             14,191.71                   851.50             15,043.21             29,234.92             70,765.08
3             14,191.71               1,754.10             15,945.81             45,180.73             54,819.27
4             14,191.71               2,710.84             16,902.55             62,083.28             37,916.72
5             14,191.72               3,725.00             17,916.72             80,000.00             20,000.00
 
 
 
 
 
 
   

Workbook in Math of Investments    55 
 
CHAPTER 8 EXERCISES 
I. Finding the unknown. 
  a. Straight‐line method 
Cost  Freight  Set‐up  Total cost  Salvage  Useful life  Total  Annual 
cost  value  Dep’n  Dep’n 
1.   45,000  150  500  ________  3,500  10 years  ________   
2. 158,200  0  1800  ________  20,000  ________  ________  14,000 
3.   88,600  625  2,500  ________  9,000  7 years  ________   
4.     5,600  210  54  ________  600  ________  ________  658 
 
  b. Sum‐of‐the‐Years’ Digits Method 
    Depreciation rate fraction 
Useful life  SYD  Year 1  Year 3  Year 5 
1.     5 Years  ________  ________  ________  ________ 
2.     7 years  ________  ________  ________  ________ 
3.   10 years  ________  ________  ________  ________ 
 
  c. Declining‐balance method 
Years  Straight line rate  Multiple  Declining Balance 
Rate 
1.           4  ________  125%  ________ 
2.           5  ________  200%  ________ 
3.          10  ________  150%  ________ 
4.           9  ________  125%  ________ 
5.           6  ________  200%  ________ 
 
  d. Units‐of‐Production Method 
Cost  Salvage Value  Units of Useful life  Dep’n per Unit 

1.      45,000     5,000  250,000 units  ________ 


2.    274,000  34,000   60,000 kms  ________ 
3.    155,000    2,000      15,000 hours  ________ 
4.        8,900       250    500,000 gallons  ________ 
5.        3,900           0  160,000 copies  ________ 
 
 
 
 
 
 
 

Workbook in Math of Investments    56 
 
II. Preparing a Depreciation Schedule 
 
1.  MB  purchased  a  new  machine  worth  P570,000.  Freight  charges  were  P4,700  and 
  installation amounted to P5,000. Estimated useful life is 5 years with a salvage value  
of P20,000. Prepare a depreciation schedule using straight‐line method. 
 
 
 
2.  A  new  unit  of  production‐line  machinery  was  purchased  for  P445,000.  Estimated 
useful   life  of  the  machine  is  6  years  and  a  trade‐in  value  of  P25,000.  Prepare  a 
depreciation schedule using sum‐of‐the‐years’ digit method. 
 
 
 
3.  Demetrius Air Service bought a fleet of helicopter for P38,600,000. It is expected to 
have  a  useful  life  of  4  years  and  a  trade‐in  value  of  P7,000,000.  Prepare  a 
depreciation schedule using the 150% declining‐balance method. 
 
 
 
4.   A meat truck was purchased by Cleveland Marketing worth P545,000. It is expected 
to have a useful life of 75,000 kms. Scrap value was set at P75,000. If the truck was 
driven   for  the  following  number  of  kilometers  per  year,  prepare  a  depreciation 
schedule using   the units‐of‐production method. 
 
  Year  1  2  3  4  5 
  Kms Driven  12,500  18,300  15,900  19,100  12,400 
   
 
5.   Anabel  purchased  an  equipment  for  P24,000.  Estimated  useful  life  is  4  years  and 
salvage  value is P3,000.  Assume that the effective interest rate is 5%.  
  (a) Construct a depreciation schedule using the annuity method. 
  (b) Construct a depreciation schedule using the sinking fund method. 
 
 
 
 
 
 
 
     
   

Workbook in Math of Investments    57 
 
Chapter 9 

STOCKS, BONDS AND MUTUAL FUNDS 
   
  Corporations need resources  to finance their various business  operations.  These resources 
can be in the form of debts, stocks and retained earnings. Commonly used though more expensive is 
the issuance of additional shares of stock. 
 
Definition of terms: 
 
Stocks – represent ownership and rights of the holders in a corporation. 
 
Certificate of stock – is a formal written evidence of the holder’s ownership of one or more shares 
and is a convenient instrument for the transfer of title. 
 
Scripless  Trading  –  is  the  shift  from  physical  transfer  of  stock  certificates  to  electronic  book‐entry 
securities transactions. 
 
Dividends – are earnings distributed to shareholders of the corporation on a pro‐rata basis. It may 
be in the form of cash or shares. 
 
For discussion purposes, a stock quotation table is from the Philippine Stock Exchange is shown in 
the next page and each column entry in the stock quotation is explained as: 
 
a  &  b.  52  Weeks  High  and  Low.  The  highest  and  lowest  traded  prices  of  a  stock  for  the  past  52 
weeks. 
c. Name. The name of the listed company. 
d. EPS. Earnings per share, calculated by dividing after‐tax income by number of shares outstanding. 
e. Prev Close. The closing price of the previous trading day. 
f. Open. The opening price of the stock for the day. 
g. High. The highest traded price of a stock during a specific trading period. 
h. Low. The lowest traded price of a stock during a specific trading period. 
i. Close. The closing price of the trading day. 
j. Volume. The total number of shares traded during a given period. 
k. Value. The amount of transactions in pesos traded for a period. 
l.  %  change.  Calculated  as  (i‐e)/e.  A  negative  value  indicates  that  the  closing  price  for  the  day  is 
lower than that for the previous day. 
m.  PE  ratio.  Price  earnings  ratio.  Calculated  as  i/d.  It  indicates  how  much  an  investor  pays  for  a 
company’s earning power. 

Workbook in Math of Investments    58 
 
 
 
 
 

Workbook in Math of Investments    59 
 
Dividends on Preferred and Common Stock 
 
Cash  dividends  must  first  be  paid  to  preferred  shareholders  before  any  common  shareholders  are 
paid. 
 
When preferred shares are non‐participating, the stockholders receive only the fixed dividend and 
no more. 
 
When  preferred  shares  are  participating,  the  stockholders  may  receive  additional  dividends  if  the 
company does well.  
 
When  preferred  shares  are  cumulative,  any  dividends  in  arrears  must  be  paid  to  preferred 
shareholders before allocating any dividends to common shareholders. 
 
When  preferred  shares  are  non‐cumulative,  only  the  current  year’s  dividend  must  be  paid  to 
preferred shareholders before paying any dividends to common shareholders. 
 
Convertible preferred means the stock may be exchanged for a specified number of common shares 
in the future. 
 
The steps to distribute dividends on preferred and common stock follow: 
 
1.  If  the  preferred  stock  is  cumulative,  any  dividends  that  are  in  arrears  are  paid  first;  then  the 
preferred dividend is stated in pesos (no‐par) go to Step 2. When the dividend per share is stated as 
a percent (par), multiply the par value by the dividend rate. 
 
      Dividend per share (preferred) = Par value x dividend rate 
 
2. Calculate the total amount of the preferred stock dividend by multiplying the number of preferred 
shares by the dividend per share. 
 
      Total preferred dividend = number of shares x dividend per share 
 
3. Calculate the total common stock dividend by subtracting the total preferred stock dividend from 
the total dividend declared. 
 
      Total common dividend = total dividend – Total preferred dividend 
 
4.  Calculate  the  dividends  per  share  for  the  common  stock  by  dividing  the  total  common  stock 
dividend by the number of shares of common stock. 
 
      Common dividend per share =    
 

Workbook in Math of Investments    60 
 
Illustration1: FTF Corporation  has 2,500,000 shares  of common stock outstanding. If a  dividend of 
P4,000,000  was  declared  by  the  company  directors  last  year,  what  is  the  dividend  per  share  of 
common stock? 
  
, ,
  Dividend per share (common) =   =   = P1.60 per share  
, ,
 
Illustration2:  The  same  with  illustration  1,  except  that  the  corporation  has  1,000,000  shares  of 
preferred  stock  that  pay  dividend  of  P0.50  per  share.  Calculate  the  amount  due  to  preferred 
shareholders and the dividend per stock of common stock. 
 
1. Total preferred dividends = Number of shares x Dividend per share 
 
  Total preferred dividends = 1,000,000 x P0.50 = P500,000 
 
2. Dividend per share (common) =  
.
    
, , ,
  Dividend per share (common) =  = P1.40 per share 
, ,
 
Illustration 3: AFV Company has 100,000 shares of P100 par value, 6% cumulative preferred stock 
and 2,500,000 shares of common stock. Although no dividend was declared last year, a P5,000,000 
dividend  had  been  declared  this  year.  Calculate  the  amount  of  dividends  due  the  preferred 
shareholders and the dividend per share of common stock. 
 
Because  the  preferred  stock  is  cumulative,  and  the  company  did  not  pay  dividends  last  year,  the 
preferred  shareholders  are  entitled  to  the  dividends  in  arrears  and  the  dividends  for  the  current 
period. 
 
1. Dividend per share (preferred) = Par value x Dividend rate 
 
Dividend per share (preferred) = 100 x 6%  = P6.0 per share 
 
2. Total preferred dividend (per year) = Number of shares x Dividend per share 
   
Total preferred dividend = 100,000 x P6.0 x 2 = P1,200,000 
 
3. Dividend per share (common) =  
.
 
, , , ,
Dividend per share (common) = = P1.52 per share 
, ,
 
 
 
 
 

Workbook in Math of Investments    61 
 
Current Yield for a Stock 
  Current yield is a way of measuring the rate of return earned from dividends. When a stock 
does not pay dividends, there is no current yield. The current yield for a stock is determined by the 
equation: 
 
Current Yield =   
 
Illustration 4: Determine the current yield for PTP Corporation stock, which pays a dividend of P1.70 
per year and is currently selling at P34.50 per share. 
 
 
.
         Current Yield =   = .0493 = 4.93% 
.
 
Price‐Earnings Ratio of a Stock 
  Price earnings ratio measures the relationship between market price per share and earnings 
per share. This ratio reflects “buyer confidence” in a particular stock compared with the market as a 
whole. The price‐earnings ratio of a stock is expressed in the equation: 
 
Price‐earnings ratio =   
 
Illustration 5: FCA stock is currently selling at P104.75. If the company had earnings per share of P4.6 
last year, calculate the price‐earnings ratio of the stock. 
 
.
        Price‐earnings ratio =   = 29.0972 or 29:1 
.
 
  The ratio shows that investors are currently willing to pay 29 times the earnings for 1 share 
of FCA stock. 
 
 
Cost, Proceeds and Gain (Loss) on a Stock Transaction 
  The  cost  of  purchasing  stock  includes  not  only  the  purchase  price  but  also  brokerage 
commission.  Brokerage  commission  rates  are  competitive,  and  vary  from  broker  to  broker.  Rates 
range from 0.25% to 1.5%. 
 
Proceeds from selling the stock = selling price – brokerage commission 
 
Gain or Loss = Proceeds from sale – the cost of purchasing stock 
 
 
 
Illustration 6: Claviel decides to buy a stock whose market price is P10.00 and with a par value of 
P1.00. The minimum number of shares he can buy at a regular transaction is 1,000 shares. Compute 
for the required cash flows. 

Workbook in Math of Investments    62 
 
 
  Market Price Per Share        P10,000 
  Multiply by Minimum No. of Shares        1,000 
                  P10,000 
  Add: Broker’s Commission (1.5%)                150 
  Total Cash Outlay            P10,150 
 
After a year, Claviel opts to sell all the shares he previously bought. Current market price is P13.00 
per share.  Compute for the cash inflows. 
 
  Market Price Per Share        P13.00 
  Multiply by Minimum No. of Shares      1,000 
                   P13,000 
  Less: Broker’s Commission (1.5%)                195 
  Net Cash Receivable            P12,805   
 
Return on Investment     
  Return  on  investment  measures  the  total  monetary  gain  on  a  stock  for  an  investor.  It  is 
expressed as:    
 
𝐍𝐞𝐭 𝐠𝐚𝐢𝐧 𝐓𝐨𝐭𝐚𝐥 𝐝𝐢𝐯𝐢𝐝𝐞𝐧𝐝𝐬
ROI =   
𝐓𝐨𝐭𝐚𝐥 𝐜𝐨𝐬𝐭 𝐨𝐟 𝐬𝐭𝐨𝐜𝐤 𝐩𝐮𝐫𝐜𝐡𝐚𝐬𝐞
 
Illustration 7: Using the data in the previous illustration and assuming that a total of P1,300 in 
dividends was received for the year, Claviel’s ROI is calculated as follows: 
 
, , ,
ROI =   
,
 
ROI = 0.3897 or 38.97% 
 
BONDS 
  A  formal  unconditional  promise  made  under  seal  to  pay  a  specified  sum  of  money  at  a 
determinable future date, and to make periodic interest payments at a stated rate until the principal 
sum is paid. 
 
 
Cost, Proceeds and Gain (Loss) on a Bond Transaction 
 
Cost of bonds includes the purchase price, broker’s commission, taxes and other charges incurred in 
their acquisition. 
 
Gain (or loss) from a bond transaction is the difference between the proceeds from the sale and the 
cost of purchase. 

Workbook in Math of Investments    63 
 
Current Yield for a Bond 
  Current  yield  of  a  bond  is  a  simple  measure  of  the  return  on  investment  based  on  the 
current market price. When bonds are purchased at face, the current yield is equal to the coupon 
rate. Current yield for a bond is expressed as: 
 
𝐀𝐧𝐧𝐮𝐚𝐥 𝐢𝐧𝐭𝐞𝐫𝐞𝐬𝐭
Current Yield =   
𝐂𝐮𝐫𝐫𝐞𝐧𝐭 𝐦𝐚𝐫𝐤𝐞𝐭 𝐩𝐫𝐢𝐜𝐞
 
Illustration 8: Calculate the current yield for JJV Centrics bond with face of P1,000 with a coupon 
rate of 6.75%, and currently selling at a premium of 107.25. 
 
  Annual interest = Face value x Coupon rate = 1,000 x .0675 = P67.50 
  Current price = Face value x Price percent = 1,000 x 1.0725 = P1,072.50 
 
               Current Yield =   
 
.
                  Current Yield =   = .0629 = 6.29% 
, .
 
MUTUAL FUNDS 
  A mutual fund is a fund managed by an investment company. It means that someone works 
on your behalf. There are funds having high potential growth but high risk while others have lower 
potential growth but less or tolerable risk.  
   
Load mutual fund – charges a fee when you buy or sell shares. 
 
No‐load mutual fund – do not charge a fee when you buy or sell shares. 
 
Common mutual fund charges: 
Load – 0.25% to 3%;  
Early redemption fee – 1%;  
Management, advisory and distribution fees – 0.75 to 2% per annum based on the net asset value of 
the fund     
 
𝑵𝒆𝒕 𝑨𝒔𝒔𝒆𝒕 𝒐𝒇 𝒕𝒉𝒆 𝒇𝒖𝒏𝒅
𝐍𝐞𝐭 Asset Value
𝑵𝒖𝒎𝒃𝒆𝒓 𝒐𝒇 𝑺𝒉𝒂𝒓𝒆𝒔 𝑶𝒖𝒕𝒔𝒕𝒂𝒏𝒅𝒊𝒏𝒈

For illustration purposes, a sample mutual fund quotation is shown below: 
 
Fund  NAV  NET CHG  YTD % RET 
Fixed Income Fund  179.47  0.08  11.05 
Equity Fund  255.82  0.03  30.31 
Balanced Fund  135.67  0.01  21.34 
 

Workbook in Math of Investments    64 
 
Looking  at  the  equity  fund,  the  net  asset  value  for  today  is  at  P255.82  per  share.  The  fund  closed 
P0.03 higher that yesterday’s closing price, that means yesterday’s closing price is P255.79. The fund 
has a 30.31% return since January 1 of the year. 
 
Return on Investment  
  To check whether an investor made the right decision to invest in a certain fund, computing 
the return on investment from such fund can be a tool. The return on investment for a mutual fund 
depends on the increase in net asset value and on the dividends paid from the fund expressed in the 
equation: 
   
𝐄𝐧𝐝 𝐨𝐟 𝐭𝐡𝐞 𝐲𝐞𝐚𝐫 𝐍𝐀𝐕 𝐃𝐢𝐯𝐢𝐝𝐞𝐧𝐝 𝐃𝐢𝐬𝐭𝐫𝐢𝐛𝐮𝐭𝐢𝐨𝐧 – 𝐁𝐞𝐠.𝐘𝐞𝐚𝐫 𝐍𝐀𝐕
ROI =   
𝐁𝐞𝐠. 𝐘𝐞𝐚𝐫 𝐍𝐀𝐕
 
Illustration 9: A certain fund net asset value on January 1 was 104. Dividend distributions during the 
year amounted to P25 per share. Net asset value at the end of the year was set at P100. Calculate 
ROI. 

ROI 20.19%
      
 
 
 
 
     
 
 
 
 
 
   

Workbook in Math of Investments    65 
 
 

CHAPTER 9 EXERCISES 
I. Finding the unknown. 
a. Find the Preferred and Common Dividend per Share 
Common  Preferred Shares  Dividend  Arrears 
Shares  Shares  Div. or Par  Cum  Declared 
1.     5,000,000    None    P3,000,000  None 
2.   10,000,000  3,000,000  P5.50  No  25,000,000  None 
3.     8,000,000  2,000,000  P100, 6%  No  10,000,000  None 
4.     4,000,000  1,000,000  P100, 4%  Yes  14,000,000  1 year 
5.   20,000,000  4,000,000  P6.25  Yes  none  1 year 
 
b. EPS, CY Ratio, PE Ratio, Dividends and Market Price 
Earnings per  Annual Dividend  Current Price  Current Yield  Price‐Earnings 
Share  per Share  Ratio 
1.         32.95  8.00  230.63  _________  _________ 
2.           3.85  1.20  88.13  _________  _________ 
3.     _________  2.25  122.50  _________  21 
4.         24.60  8.00  _________  2.5%  _________ 
5.     _________  _________  211.88  0.7%  30 
 
c. Total Cost, Proceeds, and Gain (Loss) and ROI 
No. of  Purchase  Selling  Dividend  Buying  Total Cost  Proceeds  Gain (Loss)  ROI 
Shares  Price  Price  per  and 
Share  Selling 
Rate 
     
1.  100  142.50  169.50  5.09  1%  ________  ________  ________  ________ 
2.  900  85.87  108.75  5.44  ¾%  ________  ________  ________  ________ 
3.  500  77.63  77.63  3.11  1%  ________  ________  ________  ________ 
 
d. Annual Interest and Current Yield 
    Assume that the bonds issued by the following companies have a uniform face value      
    of P1,000. 
Coupon Rate  Annual Interest  Market Price  Current Yield 
1.         6.625  __________  91.13  __________ 
2.           9.25  __________     108  __________ 
3.             7.5  __________  125.25  __________ 
4.      11.875  __________    73.5  __________ 
5.        5.375  __________  84.38  __________ 
 
e. Mutual Fund Quotation 
FUND      NAV      NET CHG    YTD % RET 
Balance Fund    14.64      ‐0.45      11.35 
1. Give interpretation to the mutual fund quotation above. 

Workbook in Math of Investments    66 
 
II. Word Problems 
 
1.  Agile Pharmaceuticals has 300,000 shares of P100 par value, 7.5%, cumulative 
preferred stock and 5,200,000 shares of common stock. Although no dividend was 
declared last year, a P7,000,000 dividend has been declared for this year. Calculate 
the amount of dividends due the preferred shareholders and the dividend per share 
pf common stock. 

 
 
 
  2.  You purchased 650 shares of Amore, Inc. common stock at P132.75 per share. A  
    few  months  later  you  sold  the  shares  at  P153.38.  Your  stockbroker  charges  *¾  % 
    commission.  

    (a) What is to total cost of purchase?  
    (b) What are the proceeds on the sale?     
    (c) What is the gain or loss on the transaction? 
    *Selling and Buying Rate 
 

  3.  Using the answers obtained in Problem # 2 compute for the ROI of Amore, Inc. 
    Dividends received during the year =P 7.8 dividend per share 
 
 
 
4.  On  January  1,  Peso  Bond  Fund’s  total  assets  were  P350  and  total  liabilities  were 
P190. During the year the fund distributed P20 per share to investors. At the end of 
the year, total assets were P382 and total liabilities were P220. Calculate the return 
on investment.  
 
 
 
 
  5.  The Elm Industries is currently selling at P186.88 per share which pays a dividend of 
      P6.80 per share. Earnings per share for the past 12 months is P33.50.  
    (a) What is the current   yield on the stock?  
    (b) What is the price‐earnings ratio for Elm? 
 
 

Workbook in Math of Investments    67 
 
 
 
 
Appendix 1 – Present Value Interest Factor 

Workbook in Math of Investments 
 

 
68 
 
 
Appendix 2– Present Value Interest Factor for an Ordinary Annuity 

Workbook in Math of Investments 
 

 
69 
 

 
 
 
 
 
 
 
 
 
 
 
 
 
 
 
 
 
 
 
 
 
 
 
 
 
 
 
 
 
 
 
 
Appendix 3 – Future Value Interest Factor 

 
 
 
 
 
 
 
 
 
 
 
 
 
 

Workbook in Math of Investments    70 
 
 
 
Appendix 4 – Future Value Interest Factor on an Ordinary Annuity 

Workbook in Math of Investments 
 
71 

You might also like